Sei sulla pagina 1di 50

I.M.

SECHENOV FIRST MOSCOW STATE MEDICAL UNIVERSITY


_______

MEDICAL FACULTY
Division for Foreign Students with Instruction Conducted in English
_______

Department of Microbiology, Virology and Immunology

GET READY
to MICROBIOLOGY FINAL EXAM!

E. V. Budanova, N. V. Khoroshko, Yu.V. Nesvizhsky, Yeu.P. Pashkov

M O S C O W - 2015
The FINAL MICROBIOLOGY EXAM INFORMATION

The procedure of Examination consists of THREE STEPS:

 STEP 1. The Examination MCQ (Written Test). December, 21 and 22, 2015
ATTENTION! All students MUST take the first step of Examination in December.
There is ONE ATTEMPT only! NOTE: There is no re-attempt anymore!
The result of exam MCQ is placed to the Record Book Page of your Protocol Notebook. Don’t forget
to bring it to the Exam!

Aim: to test your knowledge on General Microbiology, Virology & Immunology.


Part 1. Morphology of the Microbes –10 questions (multiple choice and multiple matching)
Part 2. Physiology and Genetics of the Microbes; General Virology - 30 questions (multiple choice
and multiple matching)
Part 3. Infectology and Immunology – 20 questions (multiple choice)
Number of questions: 60
Number of MCQ variants: 20
Time for answering: 60 minutes
Answering: You will put down your answers on a separate answer sheet by filling the grids of table
with letters from A to E (only one answer for each question is required!)
Remark: use a pen not a pencil to fill in the answer sheet!
NOTE: for the examples of MCQ see below!
Marks: The results are evaluated as "passed" (A, B or C) or "non-passed" (your mark is placed to
the appropriate box in the record book page of your Protocol Notebook!)

ATTENTION: If the MCQ is not passed, the total score is lowered!!!


ATTENTION!
Students permitted to get “excellent” or “good” mark (“автомат”)
without the oral step of exam must pass the exam MCQ
with the excellent or good result respectively.
You may receive your examination “excellent” or “good” mark to the credit book
after the exam MCQ.
*****
If you are not agree with your “automat mark” you can take the oral exam
in January, 19, 2016, although your exam mark can be lowered as well!
 STEP 2. Oral Test – The Examination Case. January, 19, 2016 (see the Exam Schedule!)
Aim: to test your knowledge on Special Microbiology (Bacteriology and Virology).
The case will deal with characteristics of microbe, the causative agent of bacterial or viral infection.
Time for preparing: 15 minutes
Answering: you will put down your answers on a correct side of a double-sided card with the scheme of
answer (one side is for viral and another one – for bacterial infection pathogen).
Remark: Prior to answer one should choose the correct side of answer card.
NOTE: the example of exam case can be found in MANUAL “General and Special Virology” and this
file (see below!)
Marks: The result is evaluated as “excellent”, “good” or “satisfactory” mark or "non-passed
(unsatisfactory mark).

 STEP 3. Oral Test - The Examination Task. January, 19, 2016 (see the Exam Schedule!)
Aim: to test your ability to use and apply your knowledge on General and Special Microbiology
(Bacteriology and Virology).
The task may include:
 describing the morphology and tinctorial properties of a microbe.
NOTE: you will be supplied with the pictures of bacteria examined using brightfield
microscope or electron microphotograph of viruses (see the file “PICTURES”)

 assessing the results of antibiogram (the antibiotic susceptibility test), or


 reading and explaining the results of diagnostic immune (VN-test, IHA-test, CFT, HAI-test,
precipitin ring test, etc.) or non-immune tests (VHA-test).
Time for getting ready: 10 minutes
Answering: To get ready you may use a special answer sheet. The examiner will talk to you and assess
your oral answer.
Remark: the pictures and photographs will be included to the envelop with the task (for examples see
the file “PICTURES”. Do not forget to revise all the Protocols, Tables and Pictures in your sketchbook
before the exam!)
NOTE: the example of exam task can be found in this file.
Marks: The result is evaluated as “excellent”, “good” or “satisfactory mark” or "non-passed”
(unsatisfactory mark).
NOTE: The total mark for the Microbiology exam is the average mark for all the steps of the Exam.
TWENTY COMMON QUESTIONS ABOUT THE EXAM
1. What do we need to take an exam? No. Please do not mark question sheets and do not
On the examination you are required to have a read the marks possibly made before: they could
credit book. You also need to have your protocol be wrong.
notebook with the record book page, and a pen.
12. Can I use deleting fluid/tippex?
2. Do we need to wear medical uniform on exam? On MCQ and exam case you should not use tippex:
Yes, but a cap is not obligatory. it is better to cross out mistakes neatly and write
the answer again.
3. Do we need to bring lecture notes to exam?
No. They are not required. 13 Are there the constructions like EXCEPT in
MCQ?
4. Can we use telephone, etc. on exam? No. The Exam MCQ includes multiple choice and
No. The electronic communication systems (e.g. multiple matching and there are no constructions
telephones, hands free phones, CD or MP3- like EXCEPT.
players, iPad or iPhone 4,5, 6, etc.) are
prohibited. Please, switch off all the things 14. Who does check the papers?
named above before getting inside examination Your MCQ papers are checked individually by an
room. examiner (but not by your teacher).

5. What is the pass mark for MCQ? 15. Can I take the exam with my teacher?
A student needs to get about 70% of the total No, it is not common practice. You will take the
mark (43 correct answers) to get mark ‘passed’. examination with another examiner, not with your
teacher.
6. Can we take the next steps of exam if we do not
pass the MCQ? 16. Is there any noise in the examination room?
Yes. But the TOTAL SCORE is lowered. No, if you keep a silence.

7. What is the pass mark? 17. Can I take another case/task if I do not like it?
There is no absolute pass mark, but it is better to It is not usually allowed. Prior to ask to change it,
get positive marks for all the steps of exam. shake up and think: the next one could be
worse/harder! If you still want to change the
8. Do we have to fulfil all the steps to pass the case/task, remember, that the final mark will be
exam? one point less.
No, but it is really desirable. Your final grade is
based on your total score for all three steps, so 18. Can we ask some questions to examiners before
you can do badly in one step but make up for it the exam?
by doing better on the other steps. But if you do Yes. You may ask any of your questions on the
not pass at least one of the steps, your final mark Microbiology course to the examiner during the
will be markedly decreased. pre-exam consultation (you will be informed later
about the SCHEDULE of the pre-exam
9. Do we write in pencil or ink? consultation). Do not forget to take your protocol
You can do your rough drafts in pencil, but the notebook for the consultation. Your instructors are
final answer must be in ink. also ready to answer any of your questions at the
agreed extra time.
10. Do we need draft?
You will be supplied with the sheet of paper for 19. Who will be allowed to get EXCELLENT
getting ready to answer and you will definitely MARK without ORAL EXAM?
no need draftr for MCQ and the exam case, Those students who attended all practical lessons,
because using it you will just be wasting the passed all colloquiums in due time, obtained 54 to
time. If you need an extra paper sheet to answer 60 points totally for both semesters and perform
the exam task, put your hand up and ask the the Exam MCQ with the excellent mark will be
supervisor for some more. And the draft must be allowed to get excellent mark for final
passed with the answer sheet after finishing Microbiology exam without the oral step of the
work. examination.

11. Can we write on the question paper? 20. Who will be allowed to get GOOD MARK
without ORAL EXAM?
Those students who attended all practical lessons, MCQ with the excellent or good mark will be
passed colloquiums in due time, obtained 48 to allowed to get good mark for final Microbiology
53 points for both semesters and fulfill the Exam exam without the oral examination.
The List of Microscopic Slides given on the EXAM (for microscopic examination)

Morphological groups of bacteria:


Cocci:
 Staphylococcus aureus (pure culture) - Methylene Blue or Fuchsin Staining, or Gram Stain
 Micrococcus luteus (pure culture) - Methylene Blue or Fuchsin Staining, or Gram Stain

Rod-shaped bacteria:
 Escherichia coli (pure culture) - Methylene Blue or Fuchsin Staining, or Gram Stain
 Bacillus cereus (pure culture) – Methylene Blue or Fuchsin Staining, or Gram Stain, or Endospore
Anjesky Staining

Fungi:
 Candida albicans (pure culture) - Methylene Blue or Fuchsin Staining, or Gram Stain

Special Slides:
 B. cereus and E. coli (mixture) - Gram Stain
 S. aureus and E. coli (mixture) - Gram Stain
 Klebsiella pneumoniae (capsule demonstration) - Negative Stain (Gins' Stain)
 B. cereus (endospores demonstration) - Anjesky Stain (Endospores Stain)

Dear Student!
During microscopical examination of the exam slides:
1) Examine microscopically the slide using immersion oil objective. Study the morphology and
determine staining technique.

2) Take an examination sheet and put down:


 the name of the microorganism;
 the staining technique used.

3) Let examiner to see the slide and to check your notes.


***

The List of Microscopic Preparations PICTURES included in the exam tasks

Morphological groups of bacteria:


Cocci:
 Staphylococcus aureus (pure culture) - Gram Stain
 Streptococcus pyogenes (pure culture) - Gram Stain
 Streptococcus pyogenes (in pus) - Gram Stain
 Streptococcus pneumoniaes (in patient’s specimen) - Gram Stain
 Micrococcus luteus (pure culture) - Gram Stain
 Neisseria gonorrhoeae in pus – Methylene Blue Staining
 Neisseria meningitides (pure culture) - Gram Stain

Rod-shaped bacteria:
 Escherichia coli (pure culture) - Gram Stain
 Gram-negative rods (pure culture) - Gram Stain
 Bacillus anthracis (pure culture) – Endospore Anjesky Staining
 Bacillus anthracis (in patient’s specimen) – Gram Stain
 Clostridium botulinim (pure culture) - Gram Stain
 Clostridium tetani (pure culture) - Gram Stain
 Yersinia pestis (bipolar staining) - Methylene Blue Staining
 Vibrio cholerae (pure culture) - Gram Stain

Spiral bacteria:
 Borrelia recurrentis in thick blood smear - Romanowsky-Giemsa Stain
 Treponema pallidum – Silvering Stain

Filamentous bacteria:
 Actinomyces bovis (pure culture) - Simple Stain (Fuchsin)

Fungi:
 Candida albicans (pure culture) - Gram Stain
 Candida albicans (pseudomycelium) - Gram Stain

Special Slides:
 B. cereus and E. coli (mixture) - Gram Stain
 S. aureus and E. coli (mixture) - Gram Stain
 Mycobacterium tuberculosis in sputum - Acid-Fast Stain (Ziehl-Neelsen Stain)
 Mycobacterium tuberculosis (cord-factor, microculture method) - Acid-Fast Stain
 Klebsiella pneumoniae (capsule demonstration) - Negative Stain (Gins' Stain)
 B. cereus (endospores demonstration) - Anjesky Stain (Endospores Stain)
 Corynebacterium diphtheriae (volutin granules demonstration) - Neisser' Stain
 Mycobacterium tuberculosis – Auramin (fluorescent dye) staining
 Leptospira interrhogans – Dark field microscopy picture
 Treponema pallidum – Dark field microscopy picture
 Treponema pallidum fluorescence – IF-test
 Rod-shaped bacteria fluorescence – IF-test
 Bacillus anthracis fluorescence – IF-test
 E.coli electron microphotograph
 Chlamydia trachomatis fluorescence in vaginal specimen – IF-test

Staining techniques used to characterize tinctorial properties of bacteria:


Simple Staining Procedures:
 Fuchsin Stain (red color)
 Methylene Blue Stain (blue color)

Differential Staining Procedures:


 Gram Stain (red color of gram-negative bacteria, purple color of gram-positive bacteria)
 Acid-Fast Stain (Ziehl-Neelsen Stain) (red color of acid fast bacteria against blue background)
 Romanowsky-Giemsa Stain (Eosin - Methylene Blue Stain) to reveal spirochetes (Borrelia sp.)
in blood smears (Borrelia are demonstrated as blue or violet spiral bacteria among pink stained
erythrocytes
 Silvering (Morozov’s Staining) – T.pallidum is observed as brown stained cells against yellow
background

Structural Staining Procedures:


 Negative Stain (Gins' Stain) to detect capsule of bacteria (capsule remain colorless)
 Anjesky Stain to demonstrate endospores of bacteria (endospores are stained red, vegetative cells
become blue)
 Neisser' Stain to show volutin granules of bacteria (brown or dark blue color of volutin granules,
yellow color of cell)
VIRUSES (electron microphotographs):
 Rabies virus (bullet-shaped virus)
 Cytomegalovirus inclusion bodies in the infected cells (the ‘owl-eye’-phenomenon)
 Budding process of enveloped virus
 Virus-infected cells fluorescence – IF-test
TOPICS for the EXAMINATION on MICROBIOLOGY

Academic year 2013/2014 - 2014/2015

MORPHOLOGY of the MICROBES


1. Principles of classification of the microbes.
2. Classification of bacteria.
3. Classification of fungi. Structure of fungal cell.
4. Classification of Protozoa. Morphological features of some protozoa: causative agents of a malaria,
toxoplasmosis, leishmaniasis.
5. Bacterial cell structure and its difference from eukaryotic cell structure. Chemical structure of cell
wall of Gram-positive and Gram-negative bacteria.
5. Tinctorial properties of bacteria. Staining techniques. Complex stains of bacteria. Application of
complex stains. Mechanism and procedure of the Gram staining. Mechanism and stages of Acid-
Fast Staining (the Ziehl-Neelsen staining).
6. Methods of microscopic examination (brightfield, darkfield, fluorescent and phase-contrast
microscopy). Brightfield microscopy under immersion objective.

PHYSIOLOGY of the MICROORGANISMS


1. Types of bacterial metabolism. Bacterial nutritional requirements. Mechanisms of transport and
movement of materials across bacterial membranes.
2. Types of «respiration» of bacteria (energy metabolism).
3. Enzymes of bacteria. Classification. Differential diagnostic media in identification of bacteria.
4. Bacterial growth. Generation time. The dynamics of bacterial growth in a liquid culture media.
5. Principles of cultivation of bacteria. Principles and methods of an anaerobes cultivation.
6. Microbiological culture media. Specific requirements. Classification of media by composition and
consistence. Application of media based on bacterial metabolism.
6. Pure culture of bacteria. Mechanical dissociation of microbial cells and biological bacterial
attributes. Inoculation techniques for bacterial pure culture isolation.
6. Bacteriological examination. Identification of bacteria.

GENERAL VIROLOGY
1. Viruses. Distinguishing characteristics.
2. Classification of viruses. Viral structure.
3. Types of viral replication. Stages of viral infection.
4. Principles of cultivation of the viruses. Isolation of the viruses in animals, embryonated eggs and in
tissue cultures.
5. Bacteriophages. Structure. Reproduction.
5. Virulent and temperate bacteriophages. Lysogeny.
6. Bacteriophages. Application in microbiology, medicine and biotechnology.

PRINCIPLES of BACTERIAL GENETICS


1. Bacterial genome. Genotype and phenotype.
2. Mutations and modifications in genome (genetic alteration).
3. Genetic recombination of bacteria.
4. Mechanisms of the genetic information transfer in bacteria (transformation, transduction and
conjugation).
5. Plasmids. Properties and functions of plasmids. Types of bacterial plasmids.
6. Plasmid profile of bacteria. Restriction analysis. DNA-hybridization tests.
7. Transposable genetic elements: insertion sequences and transposons.
8. Gene amplification. Polymerase chain reaction (PCR). Using for identification of microbes.
9. Genetic engineering and biotechnology - the aims and application. Progress and current research.
MICROBES in the ENVIRONMENT
1. Human normal flora. Composition and functions.
1. Normal microflora of human organism. Intestinal microflora.
2. «Eubiosis» and «dysbiosis». Dysbacteriosis, reasons for its development. Eubiotics: drugs for
restoration of normal microflora.
3. Microbial ecology. Microflora of air and water. Methods of analysis.
4. Microflora of water. Evaluating water quality: the total number of microbes, coli-titer and coli-
index.
5. Microflora of air. Examining of air contamination. Quality characteristics. Methods of
examination: sedimentation and aspiration methods.
6. Control of microorganisms. Aseptic technique. Sanitizaion. Disinfection. Sterilization.
7. Sterilization (methods and equipment).

ANTIBIOTICS and CHEMOTHERAPY


1. Antibiotics and chemotherapeutic agents. Antimicrobial effects: microbicidal and microbistatic
effects.
2. Antibacterial agents. History of discovery. Classification according to chemical structure, spectrum
and mechanism of action. Sources of antibiotic production and ways of manufacturing.
3. Antibiotics. Side effects. Choosing the best chemotherapeutic agent.
4. Antibiotic resistance: natural and acquired. Cross resistance.
5. Methods of determination of bacterial sensitivity to antibiotics: the diffusion methods and MIC-
test. Antibiogram. Interpretation.

INFECTION and BASIC PRINCIPLES of IMMUNOLOGY


1. Infection. Infectious process. Infectious disease. Elements essential for the occurrence of infectious
disease.
2. Types of infectious diseases. Bacteriemia. Sepsis. Toxemia.
3. Infectious disease. Specific characteristics and stages (periods) in the development.
4. Mechanisms of pathogenicity and virulence of bacteria. Virulence factors. Determination of
microorganisms’ virulence.
5. Exotoxins and endotoxins of bacteria. Their nature. Specific characteristics. Role in the
development of infectious diseases.
6. Immunity. Innate immunity. Natural and acquired immunity. Active and passive immunity.
7. Host resistance. Nonspecific factors of host defence.
8. Complement. Structure and functions. Role in host defence. Classical and alternative pathways of
complement activation.
9. Phagocytosis and phagocytes. Stages and mechanisms of phagocytosis. Incomplete phagocytosis.
10. Antigens: their nature, properties, application. Bacterial antigens.
11. Antibodies (immunoglobulins). Structure and properties of the classes of immunoglobulins.
12. Antibodies production: primary and secondary antibody responses.
13. Skin allergic tests. Mechanism and application. Microbial allergens.
14. Antibacterial, antitoxic, antiviral and antifungal immunity. Tumor immunity. Transplant
reactions. Mechanism of graft rejection.

IMMUNOLOGIC TESTS USEDfor MAKING DIAGNOSIS of INFECTIOUS DISEASES


1. Immunological (serological) diagnostic tests. Antigen-antibody interactions.
2. Agglutination test. Mechanism, components, application.
3. Indirect (passive) hemagglutination test (IHA-test). Mechanism, components (reagents),
application.
4. Incomplete antibodies. Coomb’s test. Mechanism, components (reagents), application
5. Precipitation test: mechanism, components, and procedure. Application.
6. Complement-fixation test (CF-test). Mechanism, components (reagents), application.
7. Toxin-antitoxin neutralization test. Mechanism, components, procedure and application.
8. Mechanism of IF-test (direct and indirect techniques). Application.
9. Modern immunodiagnostic tests with labeled reagents (ELISA, RIA & Immunoblotting).
Mechanism, components (reagents), application

IMMUNE BIOLOGICAL PREPARATIONS USED or TREATMENT and


IMMUNOPROPHYLAXIS
1. Acquired active and passive immunity. Preparations to induce active artificial immunity.
2. Vaccines. Classification, application. Vaccine strains. Requirements and preparation.
3. Live attenuated vaccines. Preparation and application. Merits and drawbacks.
4. Killed (inactivated) vaccines. Preparation and application. Merits and drawbacks.
5. Subunit vaccines. Preparation and application. Merits and drawbacks. Application of adjuvants.
6. Toxoids. Preparation, purification, titration, application.
7. Associated and combined vaccines. Merits and drawbacks. Application of adjuvants.
8. Synthetic peptides. Genetically engineered vaccines
9. Immunotherapy and immunoprophylaxis by using vaccines.
10. Antitoxic sera and immunoglobulins. Preparation, purification, titration. Application.
11. Serotherapy and seroprophylaxis. Complications. Prevention of complications.
12. Interferon. Nature. Preparation and manufacturing. Application.
13. Immunomodifiers and adaptogens. Immunosuppressive therapy.

SPECIAL BACTERIOLOGY and VIROLOGY


The student must know: taxonomy, classification, peculiarities of pathogens (morphology, tinctorial
and cultural properties, enzymatic activity, antigenic structure, virulence factors; resistance and
antimicrobial susceptibility; infectiveness for animals); the main epidemiological characteristics of
pathogens (source of infection, routes and modes of transmission; communicability); basic principles
of pathogenesis, main clinical symptoms, immune response and immunity of infection diseases;
specimens to be examined and microbiological diagnostic methods; immune biological preparations
for therapy, prophylaxis and diagnosis of infectious diseases.

Pathogens of Infectious Diseases for Discussion:


1. Staphylococci. Taxonomy. Distinguishing characteristics. Microbiological diagnosis. Therapy and
immunoprophylaxis of infectious diseases caused by staphylococci.
2. Streptococci. Taxonomy. Distinguishing characteristics. Microbiological diagnosis. Therapy of
infectious diseases caused by streptococci.
3. Meningococcal infections. Causative agent of cerebrospinal meningitis. Taxonomy. Distinguishing
characteristics. Microbiological diagnosis. Therapy and immunoprophylaxis.
4. Gonorrhea. Taxonomy. Distinguishing characteristics. Microbiological diagnosis. Treatment.
5. Gas gangrene. Causative agents. Taxonomy. Distinguishing characteristics. Microbiological
diagnosis. Immunotherapy and immunoprophylaxis.
6. Tetanus. Taxonomy and distinguishing characteristics of causative agent. Microbiological diagnosis.
Immunotherapy and immunoprophylaxis.
7. Botulism. Taxonomy and distinguishing characteristics of causative agent. Microbiological
diagnosis. Immunotherapy and immunoprophylaxis.
1. E.coli-causative agent of escherichiosis. Taxonomy and distinguishing characteristics. Role E. coli
in human normal flora and in the development of infectious processes. Microbiological diagnosis.
Treatment.
8. Bacterial dysentery. Causative agents. Taxonomy. Distinguishing characteristics. Microbiological
diagnosis. Treatment.
9. Typhoid and paratyphoid fevers. Causative agents. Taxonomy. Distinguishing characteristics.
Microbiological diagnosis. Treatment and immunoprophylaxis.
2. Salmonellosis. Causative agents. Specific characteristics of pathogens. Sources of infection. Routes
and modes of transmission. Methods of microbiological diagnosis. Treatment.
10. Cholera. Causative agent. Taxonomy. Distinguishing characteristics. Microbiological diagnosis.
Treatment and immunoprophylaxis.
11. Yersinia enterocolitica. Taxonomy. Distinguishing characteristics. Microbiological diagnosis.
Treatment.
12. Plague. Causative agent. Taxonomy. Distinguishing characteristics. Microbiological diagnosis.
Treatment and immunoprophylaxis.
13. Brucellosis. Causative agents. Taxonomy. Distinguishing characteristics. Microbiological
diagnosis. Treatment and immunoprophylaxis.
14. Anthrax. Causative agent. Taxonomy. Distinguishing characteristics. Microbiological diagnosis.
Treatment and immunoprophylaxis.
15. Tularemia. Causative agent. Taxonomy. Distinguishing characteristics. Microbiological
diagnosis. Treatment and immunoprophylaxis.
16. Diphtheria. Causative agent. Taxonomy. Distinguishing characteristics. Differentiation of
causative agent of diphtheria and diphtheroids (nonpathogenic corynebacteria). Microbiological
diagnosis. Antitoxic immunity: methods of determination. Treatment and immunoprophylaxis.
17. Whooping cough. Causative agent. Taxonomy. Distinguishing characteristics. Microbiological
diagnosis. Treatment and immunoprophylaxis.
18. Tuberculosis. Causative agents. Taxonomy. Distinguishing characteristics. Microbiological
diagnosis. Treatment and immunoprophylaxis.
19. Leprosy. Causative agent. Taxonomy. Distinguishing characteristics. Microbiological diagnosis.
Treatment.
20. Syphilis. Causative agent. Taxonomy. Distinguishing characteristics. Microbiological diagnosis.
Treatment.
21. Borrelia. Taxonomy. Distinguishing characteristics. Classification of disease caused by
Borrelia. Microbiological diagnosis. Treatment.
22. Leptospirosis. Causative agent. Taxonomy. Distinguishing characteristics. Microbiological
diagnosis. Treatment and immunoprophylaxis.
23. Rickettsia. Taxonomy and classification. Distinguishing characteristics.
24. Epidemic typhus. Causative agent. Taxonomy. Distinguishing characteristics. Brill-Zinsser
disease. Microbiological diagnosis. Treatment and immunoprophylaxis.
1. Q-fever. Causative agent. Taxonomy. Distinguishing characteristics. Microbiological diagnosis.
Treatment and immunoprophylaxis.
2. Legionnaires’ disease. Causative agent. Taxonomy. Distinguishing characteristics. Microbiological
diagnosis. Treatment and immunoprophylaxis.
3. Chlamydia. Taxonomy. Distinguishing characteristics. Classification of diseases caused by
Chlamydia. Microbiological diagnosis. Treatment.
4. Mycoplasmas. Taxonomy. Distinguishing characteristics. Microbiological diagnosis of diseases
caused by Mycoplasma. Treatment.
5. Actinomycetes. Taxonomy. Distinguishing characteristics. Microbiological diagnosis. Treatment.
6. Malaria. Causative agents. Taxonomy. Distinguishing characteristics. Specific cycle of
reproduction. Epidemiology. Microbiological diagnosis. Treatment.
7. Toxoplasmosis. Causative agent. Taxonomy. Distinguishing characteristics. Microbiological
diagnosis. Treatment.
8. Leishmaniasis. Causative agents. Taxonomy. Distinguishing characteristics. Microbiological
diagnosis. Treatment.
9. Amebiasis. Causative agent. Taxonomy. Distinguishing characteristics. Microbiological diagnosis.
Treatment.
25. Classification of Fungi. Morphology. Distinguishing characteristics. Role in the development of
human infectious diseases. Candida. Microbiological diagnosis. Treatment.

SPECIAL VIROLOGY
1. Smallpox virus. Taxonomy. Distinguishing characteristics. Diagnostic laboratory tests.
Immunoprophylaxis.
2. Herpes viruses. Taxonomy. Distinguishing characteristics. Diagnostic laboratory tests.
Immunoprophylaxis and treatment.
3. Chickenpox viruses. Taxonomy. Distinguishing characteristics. Diagnostic laboratory tests.
4. Hepatitis B, C, D viruses. Taxonomy. Distinguishing characteristics. Epidemiology. Virus carrier
state. Diagnostic laboratory tests. Immunoprophylaxis.
5. Arthropod-borne-viruses. Taxonomy. Classification. Distinguishing characteristics. Diagnostic
laboratory tests for diagnosing of diseases caused by Arboviruses. Immunoprophylaxis.
6. Hemorrhagic fever caused by viruses. Classification. Causative agents. Taxonomy. Distinguishing
characteristics. Diagnostic laboratory tests. Immunoprophylaxis and treatment.
7. Rubella virus. Taxonomy. Distinguishing characteristics. Diagnostic laboratory tests.
Immunoprophylaxis.
8. Measles. Causative agent. Taxonomy. Distinguishing characteristics. Diagnostic laboratory tests.
Immunoprophylaxis.
9. Mumps. Causative agent. Taxonomy. Distinguishing characteristics. Diagnostic laboratory tests.
Immunoprophylaxis.
10. Influenza viruses. Taxonomy. Distinguishing characteristics. Diagnostic laboratory tests.
Immunoprophylaxis and treatment.
11. Acute respiratory viral infections (ARVI). Causative agents. Classification. Taxonomy.
Distinguishing characteristics. Diagnostic laboratory tests. Immunoprophylaxis and treatment.
12. Rabies. Causative agent. Taxonomy. Distinguishing characteristics. Diagnostic laboratory tests.
Immunoprophylaxis.
13. Poliomyelitis. Causative agents. Taxonomy. Distinguishing characteristics. Diagnostic laboratory
tests. Immunoprophylaxis.
14. Enteroviruses. Classification. Taxonomy. Distinguishing characteristics. Diagnostic laboratory tests
for diagnosing of infectious diseases caused by enteroviruses.
15. Hepatitis A and E viruses. Taxonomy. Distinguishing characteristics. Diagnostic laboratory tests.
Immunoprophylaxis.
16. Oncogenic viruses. Taxonomy. Classification. Distinguishing characteristics.
17. HIV-viruses. Taxonomy. Distinguishing characteristics. Diagnostic laboratory tests.
Immunoprophylaxis and treatment.
18. Slow viruses and prions. Taxonomy. Distinguishing characteristics. Infections caused by slow
viruses. Clinical findings.

BASIC PRINCIPLES of CLINICAL MICROBIOLOGY.


1. Clinical microbiology. Main objectives.
2. «Opportunistic infections» (infectious diseases caused by facultative pathogens). Causative agents.
3. Abscesses and other diseases caused by pus-producing bacteria. Etiology, epidemiology,
pathogenesis and clinical manifestations.
4. Pyogenic inflammatory infectious processes. Microbiological diagnostic methods.
5. Collecting, storage and transport of clinical specimens. Bacteriological examination of sputum,
blood and urine.

UNIT 1. MORPHOLOGY of the E if all are correct


MICROBES
1. Which of the following are structural
The Task . Choose the appropriate peculiarities of Prokaryotes:
answer: 1. They have 70S ribosomes
2. They have nucleoid
A if only 1, 2 and 3 are correct 3. They may possess LPS
B if only 1 and 3 are correct 4. They lack mitochondria
C if only 2 and 4 are correct 2. The main features of bacterial LPS are
D if only 4 is correct which of the following:
1. It is an endotoxin 1. They contribute to adhesion to the
2. It is heat-labile cells
3. It is an O-antigen 2. They may take part in gene transfer
4. It contains peptidoglycan 3. They are receptors for
3. Which of the following microbes do not bacteriophages
possess cellular structure? 4. They are antigens
1. Viruses
2. Viroids 10. Which of the following microscopic
3. Prions techniques can be used to determine
4. Mycoplasmas bacterial motility?
4. Point out the distinguishing characteristics 1. Gram-stained smears in bright-field
of Protozoa: microscopy
1. They are eukaryotes 2. Dark-field microscopy
2. They possess 80 S ribosomes 3. Electron microscopy
3. They may form cysts 4. Phase-contrast microscopy
4. They may reproduce by binary fission 13. Which of the following are the
5. Point out the distinguishing characteristics components of bacterial LPS?
of Fungi: 1. Lipid A
1. They have no peptidoglycan 2. Peptidoglycan
2. They possess the cellular structure 3. O-specific polysaccharide chain
3. They contain sterols in the 4. Porins
cytoplasmic membrane 14. Which of the following microbes have
4. They are eukaryotes peptidoglycan in their cell wall?
6. Point out the wall-deficient bacteria: 1. Gram-positive bacteria
1. Rickettsia 2. Actinomycetes
2. Mycoplasmas 3. Gram-negative bacteria
3. Chlamydia 4. Fungi
4. L-forms 15. The term «tinctorial properties» refers to
7. Point out the acid-fast bacteria: as:
1. Staphylococci 1. resistance to detergents
2. Mycoplasmas 2. susceptibility to antibiotics
3. Streptococci 3. susceptibility to bacteriophages
4. Mycobacteria 4. attitude towards complex staining
8. Point out Eukaryotes: 16. Point out the differential staining
1. Protozoa techniques:
2. Eubacteria 1. Acid-fast stain
3. Fungi 2. Methylene blue staining
4. Prions 3. Gram-stain
9. Which of the following microbes belong to 4. Carbol fuchsin staining
Fungi? 17. Which of the following are the pathogens
1. Mycoplasmas that lack their cell wall due to exposure to
2. Actinomycetes antibiotics?
3. Mycobacteria 1. Candida
4. Candida 2. Mycoplasma
10. Which of the following components of 3. Plasmodium
Mycobacterium cell wall make it resistant 4. L-forms
to acids and some other chemicals? 18. Which of the following characterize the
1. Porins cell wall structure of Gram-negative
2. Mycolic acid bacteria? They
3. LPS 1. contain peptidoglycan
4. Waxes 2. have LPS
11. The main functions of bacterial pili 3. possess an outer membrane
(fimbriae) are which of the following: 4. contain teichoic acids
19. Which of the following are the features of 1. Gram-positive bacteria
Gram-positive bacteria cell wall? It: 2. Actinomycetes
1. contains peptidoglycan 3. Gram-negative bacteria
2. may be acid-fast 4. Fungi
3. has teichoic acids 28. Which of the following statements
4. possesses LPS concerning the cell wall structure of Gram-
20. The main features of bacterial LPS are negative bacteria are true?
which of the following: It 1. It contains peptidoglycan
1. is an endotoxin 2. It has LPS
2. is heat-stable 3. It possesses outer membrane
3. is an O-antigen 4. It contains teichoic acids
4. consists of capsomeres 29. The cytoplasmic membrane of bacteria
21. Which of the following factors may takes part in formation of:
stimulate aerobic bacteria to form 1. mitochondria
endospore? 2. mesosomes
1. Changes of optimal temperature 3. ribosomes
2. Lacking of oxygen 4. periplasmic space
3. Nutritional deficiency 30. Point out the distinguishing characteristics
4. Human organism inoculation of Protozoa:
22. The distinguishing characteristics of 1. They are eukaryotes
Protozoa are which of the following: They 2. They possess 80 S ribosomes
1. are eukaryotes 3. They may form cysts
2. may reproduce by binary fission 4. They lack mitochondria
3. may form cysts 31. Which of the following microbes do not
4. possess 70 S ribosomes possess cellular structure?
23. Point out the characteristic features of Gram-positive 1. Viruses
bacteria cell wall. It 2. Viroids
1. contains peptidoglycan 3. Prions
2. may be acid-fast 4. Mycoplasmas
3. has teichoic acids 32. Point out the distinguishing characteristics
4. possesses LPS of Fungi:
24. Which of the following methods can be 1. They have no peptidoglycan
used to determine the capsule in bacterial 2. They possess the cellular structure
pure culture? 3. They contain sterols in the
1. Neisser’s stain cytoplasmic membrane
2. Anjesky’s stain 4. They are eukaryotes
3. Ziehl-Neelsen stain 33. Which of the following microbes belong
4. Gins’ stain to Fungi?
25. Bacterial endospores can be detected by 1. Mycoplasmas
which of the following staining procedures: 2. Actinomycetes
1. Silvering stain (Morozov’s 3. Mycobacteria
stain) 4. Candida
2. Romanowsky-Giemsa stain 34. Point out the differential staining
3. Gins’ stain (negative staining) techniques:
4. Anjesky’s stain 1. Acid-fast staining
26. Point out necessary reagents for Gram 2. Methylene blue staining
stain procedure: 3. Gram-staining
1. Crystal violet dye 4. Gins’ staining
.2 Alcohol solution 35. Which of the following components of
3. Iodine solution Mycobacterium cell wall make it resistant
4. Methylene blue to acids and some other chemicals?
27. Which of the following microbes have 1. Mycolic acid
peptidoglycan in their cell wall? 2. Porins
3. Waxes 46. They are obligate intracellular parasites A. C
4. LPS 47. They are incapable of independent B. V
metabolism C. B
48. They are Prokaryotes D. N
49. They can be cultured in enriched artificial
36. Which of the following microscopic culture media
techniques can be used to see bacterial 50. They possess either RNA or DNA
motility directly?
1. bright-field microscopy of a gram- Match each of the following characteristics
stained smear listed below with the appropriate viruses:
2. Dark-field microscopy 51. They possess both DNA and RNA A. Simp
3. Electron microscopy 52. They have nucleocapsid B. Com
4. Phase-contrast microscopy 53. They possess supercapsid (envelope) C. Both
37. Point out the distinguishing characteristics 54. They have cell wall D. Neit
of Protozoa: They 55. They reproduce by binary fission
1. are eukaryotes Match distinguishing characteristics listed
2. possess 80 S ribosomes below with the appropriate bacteria:
3. may form cysts 56. They are obligate intracellular parasites A. Ric
4. may reproduce by binary fission 57. They can be cultured only in animated B. Ch
38. The distinguishing characteristics of models C. Bo
Fungi are which of the following: They 58. They contain either RNA or DNA D. Ne
1. have no peptidoglycan 59. They undergo a unique cycle of
2. have cellular structure reproduction
3. contain sterols in their cytoplasmic
membrane
4. are eukaryotes Match distinguishing characteristics with the
39. Which of the following pathogens lack appropriate microbes:
their cell wall due to exposure to 60. They are filamentous-shaped A. Actin
antibiotics? 61. They are Eukaryotes B. Asco
1. Candida sp. 62. They are Gram-positive bacteria C. Both
2. Mycoplasma sp. 63. They have either DNA or RNA D. Neit
3. Plasmodium sp. 64. They are Fungi
4. L-forms Match distinguishing characteristics listed below with the
40. Bacterial endospores can be detected by appropriate bacteria:
which of the following staining procedures: 65. They are viruses A. Rick
1. Silvering Stain (Morozov’s Stain) 66. They are obligate intracellular parasites B. Chla
2. Gins’ Stain (Negative Staining) 67. They are prokaryotes C. Both
3. Romanowsky-Giemsa Stain 68. They cannot produce their own ATP D. Neit
4. Anjesky Stain 69. They undergo a unique cycle of
reproduction
Match distinguishing characteristics listed below with the
appropriate microbe:
70. They are prokaryotes A.
Match each of the following characteristics 71. They are obligate intracellular parasites B.
with the appropriate classification group of 72. They are incapable of independent metabolism C.
microbes: 73. They have capsid D.
41. They have nucleoid A. 74. They are energy-dependent parasites
42. They possess 80S ribosomes B. 75. They possess either RNA or DNA
43. They possess capsid C. Match each of the following characteristics
44. They have no mitochondria D. listed below with the appropriate
45. They have either DNA or RNA classification group of microbes:
Match distinguishing characteristics listed 76. They possess nucleoid A. Prok
below with the appropriate microbe: 77. They have 80S ribosomes B. Euk
78. They have no mitochondria C. 1. have appropriate pH
79. They possess capsid D. 2. be isotonic
80. They have histones associated with DNA 3. be sterile
Match distinguishing characteristics listed 4. contain essential nutrients
below with the appropriate classification 4. Point out the methods to measure microbial
group of fungi: concentrations in tested samples:
81. They are Prokaryotes A. 1. Spread technique
82. They belong to Eumycetes B. 2. Pour plates technique
83. They possess aseptate hyphae C. 3. Streak plates technique
84. They may produce both sexual and asexual D. 4. Serial dilution method
spores 5. Cultural properties of bacteria are which of
85. Sexual cycle of reproduction does not exist the following:
Match distinguishing characteristics listed 1. Morphology of the colony
below with the appropriate microbe: 2. Shape and size of bacterial cells
86. They are obligate intracellular parasites A. Chlamydia 3. Color of the colony
87. They have nucleoid B. Viruses 4.Staining characteristics of bacterial
88. They have nucleocapsid C. Both A and B cells
89. They possess either RNA or DNA 6. Bacteriological
D. D. Neither A no B examination consists of
Match distinguishing characteristics with the which of the following steps:
appropriate classification group of fungi: 1. Inoculation of the specimen
90. They are Prokaryotes A. Phycomycetes2. Isolation of pure culture
91. They possess aseptate hyphae B. Eumycetes 3. Identification of pure culture
92. They form sporangia C. Both A and B 4. Antibiotic susceptibility test
93. They are Fungi D. Neither7.AThe
no Bmain role in human colonization
resistance play which of the following
UNIT 2. PHYSIOLOGY of the bacteria:

MICROBES 1. Staphylococcus sp.,


2. Bifidobacterium sp.
Task . Choose the appropriate 3. Candida
answer: 4. Lactobacillus sp.
8. Point out differential nutrient media:
A if only 1, 2 and 3 are correct 1. Hiss’ media
B if only 1 and 3 are correct 2. Sugar broth
C if only 2 and 4 are correct 3. Endo agar
D if only 4 is correct 4. MPA
E if all are correct
1. Which of the following microorganisms 9. Which of the following are the passive
can «destroy» hydrogen peroxide and processes of material movement across
superoxide? membranes:
1. Facultative anaerobes 1. Simple diffusion
2. Capnophiles 2. Group translocation
3. Aerobes 3. Facilitated diffusion
4.Strict anaerobes 4. Transport against concentration
2. The proper conditions for culturing of gradient
bacteria in laboratory are which of the 10. Which of the following are the favorable
following: conditions for successful isolation of strict
1. Nutrient medium anaerobes? Use of:
2. Optimum temperature 1. oxygen-free anaerobic culture
3. Suitable atmosphere media
4. Time of exposure 2. oxygen-free transport system
3. The very important characteristics of 3. anaerobic jar
cultural media are which of the following? 4. Endo agar
They should 11. Physical methods used to control microbes are
which of the following: 3. inhibit cell division
1. Radiation 4. kill most of pathogens
2. Filtration 19. Which of the following methods can be
3. Moist heat employed to determine microbial air
4. Autoclaving contamination?
12. Which of the following drugs can be used 1. Sedimentation method
to restore the disturbed balance of normal 2. Pour plate technique
flora? 3. Aspiration method
1. Bifidumbacterin 4. Disk diffusion method
2. Bacitracin 20. Transduction is the process of:
3. Bificol 1. identification of bacteria
4. Penicillin 2. temperate bacteriophage-mediated
13. The possible mechanisms of acquired process
antibiotic resistance of pathogen are 3. formation of L-forms of bacteria
which of the following: 4. gene transfer between bacteria
1. Mutations in the chromosome 21. Polymerase chain reaction allows to:
2. Production of β-lactamase 1. identify the microbes without
3. Presence of R - plasmid culturing
4. Catalase production 2. gene transfer between bacteria
3. generate the microbial nucleic acid
samples
4. determine the biochemical properties
14. The normal balance of «quality» and of bacteria
«quantity» of the microbial species in human 22. Drugs used to restore the normal flora are
microbial flora is referred to as which of the reffered to as:
following? 1. Antibiotics
1. Dysbiosis 2. Probiotics
2. Colonization resistance 3. Antiseptics
3. Selective decontamination 4. Eubiotics
4. Eubiosis 23. Which of the following drugs can be used
15. Which of the following microorganisms to restore the disturbed balance of normal
are common in normal microbial flora of flora?
human skin? 1. Penicillin
1. Propionobacteria 2. Bifidumbacterin
2. Escherichia coli 3. Bacitracin
3. Staphylococci 4. Bificol
4. Herpesvirusis 24. Eubiotics can be administered for:
16. Nystatin belongs to which of the following 1. selective decontamination
group of antimicrobials: 2. chemotherapy
1. Tetracyclines 3. identification of eubacteria
2. Polymyxins 4. treatment of dysbacteriosis
3. Aminoglycosides 25. The members of the human intestinal
4. Polyenes normal flora are which of the following:
17. Antibacterial agents that can inhibit the 1. Bifidobacterium sp.
bacterial protein synthesis are which of 2. Escherichia coli
the following: 3. Lactobacillus sp.
1. Tetracycline 4. Mycobacterium sp.
2. Penicillin
3. Erythromycin 26. Physical methods used for sterilization of
4. Trimethoprim infectious material are which of the
18. Microbicidal agents can: following:
1. stimulate the endospore formation 1. Autoclaving
2. destroy the bacterial cell 2. Pasteurization
3. Tyndalization 4. Boiling
4. Boiling 35. Which of the following methods can be
27. Antimicrobial agents that prevent the used to determine the pathogen’s
synthesis of peptidoglycan are which of sensitivity to antibiotics?
the following: 1. The disk diffusion method
1. Tetracycline 2. Sedimentation method
2. Penicillin 3. MIC-test
3. Amphotericin B 4. Phagotyping
4. Vancomycin 36. The natural antibiotics are which of the
28. Which of the following drugs are following:
antibacterial preparations? 1. Sulfa drugs
1. Azoles 2. Polymyxins
2. Polymyxins 3. Quinolones
3. Tetracyclines 4. β -lactam antibiotics
4. Polyenes 37. Biochemical properties of bacteria can be
29. Which of the following methods can be characterized by which of the following:
used to determine the pathogen’s 1. Attitude towards aniline dyes
sensitivity to antibiotics? 2. Ability to digest proteins
1. The disk diffusion method 3. Color of the colony
2. Phagotyping 4. Sugarlytic activity
3. MIC-test 38. Transformation is the process of:
4. Sedimentation method 1. gene transfer between bacteria
30. The possible mechanisms of acquired 2. formation of L-forms of bacteria
antibiotic resistance of pathogen are which 3. uptake and assimilation of naked
of the following: DNA
1. Mutations in the chromosome 4. synthesis of pigments
2. ß-lactamase production 39. Polymerase chain reaction is usually
3. R-factor plasmids employed to:
4. Catalase production 1. transfer genes between bacteria
31. The broad-spectrum antibiotics are which 2. generate the microbial nucleic acid
of the following: samples
1. Cephalosporins 3. determine the biochemical properties
2. Polyenes of bacteria
3. Floxquinolones 4. identify the microbes without
4.Polymyxins culturing
32. Physical methods used to control microbes 40. The conjugation process requires which of
are which of the following: the following:
1. Radiation 1. temperate bacteriophage
2. Filtration 2. presence of at least one sex pilus in a
3. Moist heat donor cell
4. Chemotherapy 3. integration of viral DNA into the
33. Point out the methods for isolating of bacterial chromosome
bacterial pure culture: 4. presence of F-plasmid in a donor cell
1. Spread technique 41. Nystatin belongs to which of the following
2. Pour plate technique group of drugs:
3. Streak plate technique 1. ß-lactam antibiotics
4. Pricking technique 2. Polymyxins
34. Physical methods used for sterilization of 3. Aminoglycosides
infectious material are which of the 4. Polyenes
following: 42. Natural antibiotic resistance of a pathogen
1. Autoclaving depends on which of the following?
2. Pasteurization 1. Production of penicillinases
3. Tyndalization 2. Presence of R- plasmids
3. Environmental conditions 4. can replicate independently of
4. Lacking the target bacterial chromosome
43. Antibacterial agents that can inhibit the
bacterial protein synthesis are which of the
following: 50. The conjugation is characterized by which
1. Tetracycline of the following:
2. Penicillin 1. virus-mediated gene transfer
3. Erythromycin 2. presence of at least one sex pilus
4. Trimethoprin 3. integration of viral DNA into the
bacterial chromosome
44. Which of the following properties are 4. presence of F-plasmid
characteristics of plasmids? Plasmids are 51. Cultural properties of bacteria characterize
1. small circular molecules of DNA which of the following features:
2. temperate bacteriophages in 1. morphology of the colony
bacterial chromosome 2. shape and size of bacterial cells
3. extrachromosomal circles of DNA 3. color of the colony
4. fragment of cytoplasmic membrane 4. staining characteristics of bacterial
45. Transformation is the process of: cells
1. gene transfer between bacteria 52. Bacteriological examination consists of
2. formation of L-forms of bacteria which of the following steps:
3. integration of donor’s DNA into the 1. Inoculation of the specimen
recipient’s cell 2. Isolation of pure culture
4 synthesis of pigments. 3. Identification of pure culture
46. Which of the following statements 4. Antibiotic susceptibility test
characterize bacterial genome? 53. Which of the following microorganisms
1. It consists of bacterial chromosome are the normal inhabitants of human large
and plasmids intestine?
2. It has haploid gene composition 1. Bifidobacterium sp.
3. It is readily transferred from one 2. E. coli
bacterial cell to another 3. Lactobacillus sp.
4. It possesses DNA-associated 4. Shigella sp.,
histones 54. Point out differential culture media:
47. Lysogeny is characterized by which of the 1. Hiss’ media
following: 2. Levine EMB agar
1. integration of temperate 3. Endo agar
bacteriophage into bacterial genome 4. Sugar broth
2. lysis of bacterial cell 55. Which of the following fluids and organs
3. formation of prophage of human organism are normally free of
4. synthesis of new progeny of viruses microbes?
48. Which of the following are the steps of 1. Blood
bacteriologic examination? 2. Kidneys
1. Inoculation of specimen tested into 3. Cerebrospinal fluid
the animal organism 4. Large intestine
2. Isolation of bacterial pure culture 56. Which of the following methods can be
3. Indication of virus used to isolate pure culture of bacteria?
4. Identification of bacteria isolated 1. Spread technique
49. Point out the properties of plasmids. They: 2. Pour plates technique
1. can integrate into the bacterial 3. Streak plates technique
chromosome 4. Serial dilution method
2. carry gene, that are not essential to 57. Cultural properties of bacteria characterize
bacteria which of the following:
3. can encode the resistance to 1. Shape and size of bacterial cells
antibiotics 2. Morphology of the colony
3. Staining characteristics of bacterial 4. Strict anaerobes
cells 65. Point out the differential nutrient media:
4. Color of the colony 1. Hiss’ media
58. The possible mechanisms of acquired 2. Sugar broth
antibiotic resistance of bacteria are which 3. Endo agar
of the following: 4. MPA
1. Mutations in the chromosome 66. Eubitics can be administered for:
2. ß-lactamase production 1. selective decontamination
3. R-factor plasmids 2. chemotherapy
4. Penicillinase production 3. identification of eubacteria
59. Which of the following drugs are 4. treatment of dysbiosis
antibacterial preparations? 67. Physical methods used to control microbes
1. Bifidumbacterin are which of the following:
2. Inteferon 1. Radiation
3. Polyenes 2. Filtration
4. Tetracyclines 3. Moist heat
60. Physical methods used to make 4. Chemotherapy
microbiological glass utensil (Petri 68. The broad-spectrum antibiotics are which
dishes, tubes, pipettes, etc.) sterile are of the following:
which of the following: 1. Cephalosporins
1. Autoclaving 2. Polyenes
2. Pasteurization 3. Floxquinolones
3. Dry heat 4.Polymyxins
4. Filtration 69. Which of the following statements
61. Which of the following statements characterize the plasmids? Plasmids are
characterizing the cultural media are true? 1. small circular molecules of DNA
To be effective culture medium should 2. temperate bacteriophages in
1. have appropriate pH bacterial chromosome
2. be isotonic 3. extrachromosomal pieces of DNA
3. be sterile 4. fragment of cytoplasmic membrane
4. contain all essential nutrients 70. Acquired antibiotic resistance of a
62. Bacteriological examination consists of pathogen depends on which of the
which of the following steps: following?
1. Inoculation of the culture medium 1. Production of β-lactamase
with a specimen tested 2. Mutation in a pathogen's
2. Identification of bacteria by chromosome
morphology and tinctorial properties 3. Presence of R- plasmids
3. Identification of bacteria by cultural 4. Lacking the target
properties 71. Which of the following statements
4. Identification of bacteria by concerning the conjugation process in
biochemical properties bacteria are true?
63. Which of the following bacteria play the 1. This process depends on the presence
main role in human large intestine of at least one sex pilus
colonization resistance? 2. The donor cell should be F+ or Hfr+
1. Staphylococcus sp. 3. The donor cell must have F-plasmid
2. Bifidobacterium sp. 4. It is a virus-mediated process
3. Candida sp.
4. Lactobacillus sp. Match each of the statements listed below
64. Which of the following microorganisms with the appropriate diagnostic method:
are killed by briefest exposure to oxygen?
72. It is the method for air contamination A. Sedime
1. Capnophiles
determining B. Aspira
2. Facultative anaerobes
73. It is employed to determine minimal C. Both A
3. Microaerophiles
inhibitory concentration of antibiotic D. Neithe
74. It is also called ‘slit method’ Match each of the following characteristics
75. Anaerobic jar is employed for with the bacteria respiration type:
sampling 94. They require the presence of small oxygen A.
concentration in atmosphere B.
Match each of the following characteristic 95. They are cultured in the incubator C.
with the appropriate process of across 96. They grow only in animated models D.
membranes material movement: 97. They can detoxify superoxide
98. They can reproduce only in oxygen-free
76. It is a passive process A. Simple diffusion
atmosphere
77. Transported molecule is modified B. Facilitated diffusion
78. Permease helps to move the molecule C. Both A and B
Match the nutrient medium with its
79. Bacterial cell needs to expend energy D. Neither A nor B
characteristic feature:
99. Egg yolk-salt agar A. General-purpos
Match each of the following characteristics
100. Kligler iron agar B. Enriched mediu
given below with the appropriate nutrient
101. Endo’s agar C. Selective medium
medium:
102. Sugar broth D. Enrichment
80. It is used for the storage of specimens A. Thioglycollate medium
103. MPB E. Differential med
only B. Isotonic solution (as a
104. 10% bile broth
81. It is employed for determination of transport medium)
Match each of the following processes of
sugarlytic activity of bacteria C. Sugar broth
material
82. It is used for the isolation of strict D.1% peptone water movement across membranes with its
characteristic:
anaerobes E. Hiss' medium
105. It is a passive process A. Active t
Match each of the following characteristic 106. Transported molecule is modified B. Group t
with the process of material movement across 107. Permease helps to move the molecule C. Both A
membranes: 108. Bacterial cell needs to expend energy D. Neither
83. It is a passive process Match the characteristics with the bacteria
A. Simple diffusion
84. Transported molecule is modified respiration type:
B. Group translocation
85. Permease helps to move the molecule 109.
C. Both A and B Require the presence of small oxygen A. Str
86. Bacterial cell needs to expend energy concentration
D. Neither A nor B in atmosphere B. Mi
110. May require the presence of CO2 in C. Bo
Match each of the term listed below with its atmosphere D.
definition: 111. Can be reproduced only in oxygen-free B
87. Transformation atmosphere
A. The virus-mediated gene transfer between
88. Transduction bacteria 112. May detoxify superoxide
89. Conjugation 113.
B. The incorporation of the donor’s Must be cultivated in the anaerobic jar
supercoiled
DNA into the recipient’s cell
Match each
C. The attachment of F+ donor bacterial cell toof the following characteristics
with nutrient
F- recipient cell to transfer genetic materialmedium:
114. It is used only for the storage of specimens
D. The acquisition of new properties following A. Thio
lysogeny 115. It is used for determination of medium
sugarlytic activity of bacteria B. Endo’s a
Match the statements listed below with the 116. It is used for the isolation of strict C. Sugar br
appropriate antibiotic susceptibility test: anaerobes D. 1%
90. The test in which antibiotic-impregnated disks E. Isotonic
are used transpor
91. The test in which antibiotic is diluted in Match the antimicrobial mechanisms listed
nutrient broth below with the appropriate antibiotic group:
92. The test which is employed to identify 117. Inhibit the synthesis of peptidoglycan A. T
phagotype of bacteria 118. Inhibit the cytoplasmic membrane function B. Q
93. The test in which the lowest inhibitory 119. Inhibit the bacterial protein synthesis C. S
concentration of antibiotic can be determined 120. Inhibit the DNA gyrase D.
E. P
Match each of the term listed below with its 3. It is followed by a specific immune
definition: response in the host
121. Transformation A. Virus-mediated gene transfer between 4. It can be caused by insects
122. Transduction bacteria 5. Which of the following statements
123. Conjugation characterizing the role of the environment
B. Incorporation of donor’s supercoiled
DNA into the recipient’s cellfor the infection development are true? It
C. Attachment of F+ donor bacterial1.cell affects
to Fthe host defence and the
microbial virulence
recipient cell to transfer genetic material
D. The acquisition of new properties be the source and reservoir of
2. may
following lysogeny infection
Match the statements listed below with the 3. provides the mechanisms and routs
appropriate antibiotic susceptibility test: of microbial transmission
124.The test in which antibiotic- 4. affects the microbial pathogenicity
A. Disk diffusion method
impregnated disks are used 6. Which of the following statements
B. MIC-test
125.The test in which antibiotic is diluted in C. Both A and concerning
B endotoxins are true?
nutrient broth D. 1. They are the component of bacterial
126. The test in which the lowest inhibitory A no B cell outer membrane
concentration of antibiotic can be 2. Their immunogenicity is low
determined 3. Most of them produce similar
generalized effect in the host
UNIT 3. INFECTOLOGY and 4. They may be converted into toxoid
7. Which of the following statements
APPLIED IMMUNOLOGY characterizing toxoids are true?
Task . Choose the appropriate They are lipopolysaccharides
2. They are used for immunization
answer: 3. They are protective antibodies
A if only 1, 2 and 3 are correct 4. They have lost toxicity and retained
B if only 1 and 3 are correct antigenicity
C if only 2 and 4 are correct 8. Which of the following statements
characterizes endemic infection? It
D if only 4 is correct
1. has a world-wide spreading
E if all are correct
2. occurs regularly in a specific
population
1. Point out the elements essential for the
3. spreads rapidly involving many
occurrence of infectious process:
persons within a short period of time
1. susceptible macroorganism
4. is constantly present in a specific
2. pathogenic infectious agent
population at a low level
3. specific environmental conditions
9. Which of the following functions have both
4. opportunistic microbe
a primary and secondary lymphoid organs:
2. What are the possible outcomes of the host
1. Cellular proliferation
exposure to microbes?
2. Differentiation of lymphocytes
1. Severe acute disease
3. Cellular interaction
2. Carrier state
4. Antigen-dependent response
3. Inapparent infection
10. Converting a toxin to a toxoid:
4. No infection
1. makes the toxin more immunogenic
3. Point out the types of generalized infection:
2. decreases its toxicity
1. Septicemia
3. increases phagocytosis
2. Pyemia
4. reduces the biologic activity of the
3. Toxemia
toxin
4. Anemia
11. Which of the following statements
4. The distinguishing features of infectious
characterize haptens? They
disease are which of the following:
1. require carrier molecules in order to
1. It is caused by pathogenic microbe
be immunogenic
2. It is communicable
2. will not react with specific 19. Which of the following statements
antibodies in vitro unless characterizing the antigen-binding site of
homologous carriers are employed an Ig molecule are true? It is
3. interact with specific antibody 1. destroyed by removal of sugar
4. can stimulate secondary antibody residues
response without carriers 2. a variable fragment
12. Point out the elements essential for the 3. destroyed by papain digestion
occurrence of infectious process: 4. in the first domain of L and H chains
1. susceptible macroorganism 20. Blood from group AB donors can be
2. pathogenic infectious agent transfused to a recipient without causing a
3. specific environmental conditions transfusion reaction if:
4. opportunistic microbe 1. the recipient is A
13. The distinguishing characteristics of 2. the recipient is B
human infectious disease are which of the 3. the recipient is O
following: It 4. the recipient is AB
1. is caused by pathogenic microbe 21. In the complement fixation test, the
2. is communicable hemolytic system:
3. is followed by a specific immune 1. is the indicator system
response in the host 2. fixes complement in a positive CF-
4. can be caused by insects test
14. Which of the following statements are 3. is used to detect any free
true? Microbial virulence is: (noncomplexed) complement
1. a feature of microbial strain 4. is the diagnostic system tested
2. measured in LD50 22. Human T-cells can be distinguished from
3. a degree of pathogenicity B-cells and other cells by which of the
4. a peculiarity of microbial species following:
15. Virulence factors for bacteria spreading in 1. CD markers
human organism are which of the 2. the structure of antigen-specific
following: receptors
1. hyaluronidase 3. the formation of rosettes with sheep’s
2. lecithinase RBC
3. coagulase 4. the presence of cytoplasmic granules
4. endotoxin 23. Helper T-cells induce:
16. Which of the following statements 1. proliferation of B-cells
concerning endotoxins are true? 2. differentiation of B-cells into plasma
1. They are the components of bacterial cells
cell outer membrane 3. expansion of the pool of memory B-
2. Their immunogenicity is low cells
3. Most of them produce similar 4. immunoglobulin class switch
generalized effect in the host 24. CD4+ and CD8+ T-cells in humans differ
4. They cannot be converted into toxoid from each other by which of the
17. The nonspecific humoral factors of host following:
defense are which of the following: 1. function
1. complement 2. restriction elements encoded by the
2. phagocytosis MHC
3. α - IFN 3. production of B-cell-activating
4. normal microbial flora lymphokines
18. Converting a toxin to a toxoid: 4. proliferative response to IL-2
1. makes the toxin more immunogenic 25. In the indirect immunofluorescent test, the
2. decreases exotoxin toxicity reagents combination involved may be
3. increases phagocytosis which of the following:
4. reduces the biologic activity of the 1. Patient’s serum and fluorescein-
toxin labeled diagnosticum
2. Patient’s specimens tested, diagnostic class II molecules on both
rabbit serum and fluorescein- macrophage and B-cell
labeled anti-rabbit antibody 30. Which of the following statements
3. Fluorescein-labeled patient’s serum regarding hemolytic disease of the
and rabbit anti-human antibody newborn are correct?
4. Patient’s serum, fluorescein-labeled 1. Administration of anti-Rh globulins
rabbit anti-human antibody and to an «Rh-negative» mother soon
diagnosticum after delivery of an «Rh+» baby can
26. Which of the following is a function of suppress the induction of anti-Rh
macrophages in immune response? globulins by mother
1. Phagocytosis and digestion of 2. It is an example of type II
antigens hypersensitivity reaction
2. Optimal presentation of antigen to B- 3. The mother forms antibodies against
cells Rh antigens which she lacks
3. Optimal presentation of antigen to T- 4. If the newborn is «Rh-» and the
cells mother is «Rh+», the new fetus
4. Lymphokine releasing which induces becomes tolerant to Rh antigens
direct virus killing
31. Transplant rejection may involve which of the
following immune processes:
27. Which of the following statements 1. cell-mediated immunity
concerning anaphylactic reactions are 2. type IV delayed hypersensitivity
true? They 3. complement-dependent cytotoxicity
1. may develop in minutes and abate 4.-α- IFN releasing
within one-half hour 32. The class-specific antigenic determinants
2. may be followed by inflammatory (isotypes) of immunoglobulins are
several hours later associated with which of the following:
3. are the consequence of released 1. L chains
biological active molecules from 2. H chains
mast cells 3. variable regions
4. involve Ig E 4. constant regions
28. T-cell-mediated immune response can 33. An immunoglobulin molecule consists of
result in: which of the following:
1. formation of granuloma in internal 1. two identical light chains
organs 2. variable and constant regions on
2. anaphylaxis each chain
3. transplant rejection 3. two identical heavy chains
4. serum sickness 4. polypeptide chains divided into
29. In a response of B-cells to thymus- domains
dependent antigen, it is proposed that: 34. Which of the following are the properties
1. helper T-cells recognize the antigen of human Ig G:
in association with MHC (or HLA) 1. It can pass through placenta
class II molecules on macrophage 2. It can be cleaved by pepsin and yet
2. both carrier and haptenic remain divalent
determinants have to be present on 3. It is the main Ig of secondary immune
the same molecule for effective response
cooperation between T- and B-cells 4. It induces the formation of leukocytes
3. helper T-cells produce the 35. In the indirect immunofluorescent test the
lymphokines needed for the B-cells reagent combinations involved may be
to produce the various classes of which of the following:
immunoglobulin 1. Patient’s serum and fluorescein-
4. helper T-cells may respond to the labeled diagnosticum
same antigen and MHC (or HLA)
2. Patient’s specimens tested, diagnostic 42. Which of the following statements
rabbit serum and fluorescein- characterizing the role of the environment
labeled anti-rabbit antibody in the infection development are true? It
3. Fluorescein-labeled patient’s serum and rabbit 1. affects the host defence
anti-human antibody 2. may be the source and reservoir of
4. Patient’s serum, fluorescein-labeled infection
rabbit anti-human antibody and 3. provides the mechanisms and routes
diagnosticum of microbial transmission
36. Complement is required for lysis of: 4. may change the microbial virulence
1. bacteria by specific antibodies 43. The stages in the development of
2. bacteria by bacteriophages infectious disease are which of the
3. erythrocytes by specific antibodies following:
4. erythrocytes by lecithinase 1. Prodromal period
37. The alternative pathway of complement 2. Convalescent period
activation is characterized by which of the 3. Incubation period
following: 4. Lag period
1. activation of complement components 44. Bacterial adhesins are which of the
beyond C3 in the cascade following:
2. participation of properdin 1. coagulase
3. generation of anaphylatoxin 2. capsule
4. utilization of C4 3. endospores
38. Which of the following activate the 4. pili
alternative pathway of complement? 45. LD50 characterizes which of the
1. endotoxin following features of microbes:
2. C1 1. immunogenicity
3. yeast cell wall 2. pathogenicity
4. monomeric Ig G 3. antigenicity
39. Which component or combination of 4. virulence
components of complement could be 46. Which of the following statements
missing and still leave the remainder of the characterizing toxoids are true?
complement system capable of activation 1. They are lipopolysaccharides
by the alternative pathway: 2. They are vaccines
1. C1, 2, and 3 3. They are protective antibodies
2. C3 only 4. They have lost toxicity and retained
3. C2, 3, and 4 antigenicity
4. C1, 2, and 4 47. Which of the following functions have
40. Human T-cells can be distinguished from both a primary and secondary lymphoid
B-cells and other cells by which of the organs?
following: 1. Cellular proliferation
1. morphologic appearance 2. Differentiation of lymphocytes
2. the presence of antigen-specific 3. Cellular interaction
receptors 4. Antigen-dependent response
3. the formation of rosettes with sheep 48. An immunologic adjuvant is a substance
red blood cells that:
4. the presence of cytoplasmic granules 1. reduces antigen degradation
41. Thymus processing is not necessary for 2. reduces the toxicity of the immunogen
maturation of which of the following: 3. enhances the immune response
1. helper T-cells against the immunogen
2. B-cells 4. enhances hematopoiesis
3. cytotoxic T-cells 49. The main antibody isotype that acts
4. monocytes against bacteria in mucosa is which of the
following:
1. Ig G
2. Ig D 3. phagocytes function
3. Ig M 4. complement function
4. (secretory) sIg A 56. Immune complexes are involved in the
50. The secondary antibody response differs pathogenesis of which of the following
from the primary response by having conditions:
which of the following: 1. Farmer’s lung
1. a shorter latent period 2. Serum sicknes
2. a longer duration of antibody 3. Glomerulonephritis or systemic lupus
production erythematosus
3. a greater quantity of antibodies 4. Anaphylaxis
produced 57. Specific nonresponsiveness to self and
4. prevalent pool of Ig M antibodies other antigens can be mediated by which
51. Which of the following statements of the following:
concerning activation of B-cells are true? 1. clonal deletion of self-reactive B-cells
1. Thymus-independent antigens 2. specific suppressor T-cells
activate B-cells directly to produce 3. clonal deletion of self-reactive T-cells
Ig M antibody (early stages of maturation)
2. Most protein antigens require helper 4. rapid digestion and removal of
T-cells to induce production of antigen from the circulation
antibody by B-cells
3. Optimal collaboration between T-
and B - cells require a close contact
4. When B-cells proliferate, some 58. The administration of vaccines is not
differentiate into plasma cells while others without hazard. Which of the following
remain as memory B-cells vaccines is most likely to affect adversely
52. Immunoglobulins that activate the first an immunocompromised host?
component of complement (C1q) via the 1. Killed vaccine
Fc portion include which of the 2. Toxoid
following: 3. Subunit vaccine
1. Ig G 4. Live vaccine
2. Ig A 59. Blood group incompatibility usually
3. Ig M possesses a transfusion reaction risk,
4. Ig E because:
1. a lymphocyte reaction takes place
53. Thymus processing is not necessary for 2. the donor’s immunoglobulins react with
maturation of which of the following the recipient’s erythrocytes
cells: 3. the recipient’s T -lymphocytes will
1. helper T-cells become activated by the donor’s
2. monocytes antigens
3. cytotoxic T-cells 4. the recipient’s immunoglobulins react
4. B-cells with donor’s erythrocytes
54. The MHC (or HLA) class I and II proteins 60. Blood from group A donors can be
share which of the following features: transfused to a recipient without causing a
They both transfusion reaction if:
1. are integral membrane proteins 1. the recipient is A
2. serve to restrict responses of T-killers 2. the recipient is O
3. are codominantly expressed on cells 3. the recipient is AB
4. are expressed on the surface of all 4.the recipient is B
nucleated cells 61. The complement activation by an immune
55. Immunodeficiency can result from which complex may result in:
of the following? Defect in 1. precipitation
1. T lymphocytes developing 2. opsonization
2. bone marrow stem cells 3. release of macrophage-inhibiting
factor 1. They are both integral membrane
4. release of anaphylatoxins proteins
62. Which of the following statements are 2. They both serve to restrict responses
true? The macrophages of T-killers
1. are derived from blood monocytes 3. They are both codominantly
2. have a great diversity of form expressed on cells
3. are able to ingest and degrade 4. They are both expressed on all
microorganisms nucleated cells
4. can process and present antigens to T-
cells
68. CD8+ cells kill their targets by which of
the following processes:
1. Recognition of peptides on MHC (or
63. Helper T-cells induce: HLA) class I molecules
1. proliferation of B-cells 2. Activation of the alternative
2. differentiation of B-cells into plasma complement pathway
cells 3. Insertion of representative molecules
3. expansion of the pool of memory B- into target membranes
cells 4. Binding via Fc receptors to antibody-
4. immunoglobulin class switch coated targets
64. The secondary antibody response differs 69. Immunodeficiency disease can result from
from the primary response by having which of the following? Defect in
which of the following: 1. T lymphocytes developing
1. a shorter latent period 2. bone marrow stem cells
2. a longer duration of production of 3. phagocytes function
antibody 4. complement function
3. a greater quantity of antibody 70. Which of the following statements
produced concerning anaphylactic reactions are true?
4. primarily Ig M antibody produced They
65. CD4+ cells can: 1. may develop in minutes and abate
1. help B-cells to make antibody within one-half hour
2. process and present antigen 2. may be followed by inflammatory
3. amplify activities of T-cells several hours later
4. release IL-1 3. are the consequence of released
66. In a response by B-cells to thymus- biological active molecules from
dependent antigen, it is proposed that: mast cells
1. helper T-cells recognize the antigen 4. involve Ig E
in association with MHC (or HLA) 71. Which of the following statements are true?
class II molecules on macrophage Serum sickness occurs if:
2. both carrier and haptenic 1. anti-basement membrane antibodies
determinants have to be present on are present
the same molecule for effective 2. extreme excess of antibody develop
cooperation between B-cells 3. Ig E antibodies are produced
3. helper T-cells produce the 4. soluble immune complexes are
lymphokines needed for the B-cells to formed
produce the various classes of 72. Immune complexes are involved in the pathogenesis
immunoglobulin of which of the following:
4. helper T-cells may respond to the 1. Farmer’s lung
same antigen and MHC (or HLA) 2. Serum sickness
class II molecules on both 3. Glomerulonephritis of systemic lupus
macrophage and B-cell erythematosus
67. The MHC (or HLA) class I and II proteins 4. Anaphylaxis
share all of the following features:
73. Point out the possible mechanisms for the 4. is constantly present in a specific
development of autoimmune diseases: population at a low level
1. Alteration of self antigen so it is 80. Removing of the bursa of Fabricius from a
recognized as foreign chicken results in:
2. Leakage of sequestered self antigen 1. markedly decreased number of
3. Disorders in T-cells maturation circulating lymphocytes
4. Infection with a microorganism that 2. anemia
carries a cross-reactive antigen 3. delayed rejection of skin graft
74. Blood from group AB donors can be 4. low serum levels of antibodies
transfused to a recipient without causing a 81. An immunoglobulin molecule consists of
transfusion reaction if: which of the following:
1. the recipient is A 1. two identical light chains
2. the recipient is B 2. variable and constant regions
3. the recipient is O 3. two identical heavy chains
4.the recipient is AB 4. polypeptide chains divided into
75. What are the possible outcomes of the domains
host exposure to microbes? 82. CD8+ cells kill their targets by which of
1. Acute disease the following processes:
2. Inapparent infection 1. Activation of the alternative
3. Chronic infection complement pathway
4. Carrier state 2. Recognition of peptides on MHC (or
76. Virulence factors for bacteria spreading in HLA) class I molecules
human organism are which of the 3. Binding via Fc receptors to antibody-
following: coated targets
1. hyaluronidase 4. Insertion of representative molecules
2. lecithinase into target membranes
3. coagulase 83. In the indirect immunofluorescent test, the
4. adhesins reagents combination involved may be
77. Which of the following statements concerning which of the following:
bacterial endotoxin are true? 1. Patient’s serum and fluorescein-
1. It is a component of bacterial cell labeled diagnosticum
outer membrane 2. Patient’s specimen tested, diagnostic
2. Its immunogenicity is low rabbit serum and fluorescein-
3. It is a LPS labeled anti-rabbit antibody
4. It can be converted into toxoid 3. Fluorescein-labeled patient’s serum
78. Which of the following statements and rabbit anti-human antibody
characterizes the pathogenicity of 4. Patient’s serum, fluorescein-labeled
microbes? rabbit anti-human antibody and
1. It is a potential ability to cause diagnosticum
infectious disease 84. The alternative pathway of complement
2. It can be increased or decreased activation is characterized by which of
within the host organism the following:
3. It is the ability of microbial species to 1. activation of complement components
injure the host tissue beyond C3 in the cascade
4. It depends on environmental 2. participation of properdin
conditions 3. generation of anaphylatoxin
79. Which of the following statements 4. utilization of C4
characterizes endemic infection? It
1. has a world-wide spreading
2. occurs regularly in a specific
population 85. Which components of complement could
3. spreads rapidly involving many persons be missed and still leave the remainder of
within a short period of time the complement system capable of
activation by the alternative pathway: 3. Disorders in T-cells maturation
1. C1, 2, and 3 4. Infection with a microorganism that
2. C3 only carries a cross-reactive antigen
3. C2, 3, and 4 93. The CF-test is considered to be «positive»,
4. C1, 2, and 4 if:
86. Which of the following statements are 1. the complete hemolysis occurs
true? The macrophages 2. complement fixes to hemolytic system
1. are derived from blood monocytes 3. the specific antibodies inhibit the
2. have a great diversity of form virus ability to agglutinate erythrocytes
3. are able to ingest and degrade 4. the red «button» is formed in the well
microorganisms 94. The color test in the cell culture is
4. can process and present antigens to considered to be «positive», if
T-cells 1. the virus cytopathic effect is
87. CD4+ cells can: neutralized by specific antibodies
1. help B-cells to make antibody 2. the virus causes damage of cells in
2. process and present antigen the cell culture
3. amplify activities of T-cells 3. the virus- inoculated cell culture
4. release IL-1 remains alive after specific
88. Which of the following statements antibodies adding
concerning anaphylactic reactions are 4. the color of indicator is not changed
true? They 95. Which of the following statements
1. may be followed by inflammation regarding hemolytic disease of the
several hours later newborn are correct?
2. are the consequence of released biological 1. Administration of anti-Rh globulins
active molecules from mast cells to an «Rh-negative» mother soon
3. involve Ig E after delivery of an «Rh+» baby can
4. may develop in minutes and abate suppress the induction of anti-Rh
within one-half hour globulins by mother
89. Which of the following activates the 2. It is an example of type II
alternative pathway of complement? hypersensitivity reaction
1. endotoxin 3. The mother forms antibodies against Rh
2. C1 antigens which she lacks
3. yeast cell wall 4. If the newborn is «Rh-» and the
4. monomeric Ig G mother is «Rh+», the new fetus
90. A man has blood group AB. From whom becomes tolerant to Rh antigens
he may accept blood without a 96. The MHC (or HLA) is encoded by which of the
transfusion reaction risk? From following: Genes that
1. the group A donor 1. encode transplantation antigens
2. the group B donor 2. regulate immune responsiveness
3. the group AB donor 3. encode class I and class II antigens
4. the group O donor 4. encode immunoglobulins
91. The non-specific factors of the host 97. Which of the following statements
defense against viral infections are which concerning serum sickness due to
of the following: administration of horse antitoxin for
1. Serum inhibitors passive protection in human are true?
2. T-lymphocytes Serum sickness is characterized by
3. Interferons 1. production of antibodies against
4. Ig G foreign horse antibody
92. Point out the possible mechanisms for the 2. delayed onset in 10-14 days
development of autoimmune diseases: 3. deposition of antigen-antibody
1. Alteration of self antigen so it is complexes at various sites in the host
recognized as foreign 4. cell-mediated immune response
2. Leakage of sequestered self antigen
98. Whichof the following statements 103. The administration of vaccines is not
regarding hemolytic disease of the without hazard. Which of the following
newborn are correct: vaccines is least likely to affect adversely
1. Administration of anti-Rh globulins an immunocompromised host?
to an «Rh-negative» mother soon 1. Killed vaccine
after delivery of an «Rh+» baby can 2. Subunit vaccine
suppress the induction of anti-Rh 3. Toxoid
globulins by mother 4. Live vaccine
2. It is an example of type II 104. Point out the first immunoglobulin
hypersensitivity reaction synthesized by the fetus:
3. The mother forms antibodies against 1. Ig A
Rh antigens which she lacks 2. Ig E
4. If the newborn is «Rh- « and the 3. Ig G
mother is «Rh+», the fetus becomes 4. Ig M
tolerant to Rh antigens 105. Which of the following statements
99. Which of the following statements concerning serum sickness due to
characterizing toxoids are true? administration of horse antitoxin for
1. They are lipopolysaccharides passive protection in human are true?
2. They are used for immunization Serum sickness is characterized by
3. They are protective antibodies 1. production of antibodies against
4. They have lost toxicity and retained foreign horse antibody
antigenicity 2. delayed onset in 10-14 days
100. Human T-cells can be distinguished from 3. deposition of antigen-antibody
B-cells and other cells by which of the complexes at various sites in the host
following: 4. cell-mediated immune response
1.CD markers 106. The main antibody isotype that acts
2. the structure of antigen-specific against bacteria in mucosa is which of the
receptors following:
3. the formation of rosettes with sheep’s 1. Ig G
red blood cells 2. Ig D
4. the presence of cytoplasmic granules 3. Ig M
101. Intentional protection of a future 4. Ig A
newborn against Rh disease involves which 107. Which of the following statements
of the following? concerning activation of B-cells are true?
1. Active immunization of the mother to produce 1. Thymus-independent antigens
transplacental Ig G activate B-cells directly to produce
2. Active immunization of the mother with Rh Ig M antibody
antigen to induce antibody 2. Most protein antigens require helper
3. Passive immunization of the newborn to T-cells to induce production of
remove the Rh antigen antibody by B-cells
4. Passive immunization of the mother 3. Optimal collaboration between T-
to remove Rh antigen and B- cells require their close
102. Blood group incompatibility usually possess contact
a transfusion risk because: 4. When B-cells proliferate, some
1. the donor’s immunoglobulins react differentiate into plasma cells while
with recipient’s erythrocytes others remain as memory B-cells
a mixed lymphocyte reaction takes place 108. The stages in the development of
the recipient’s T lymphocytes will infectious disease are which of the
become activated by the donor’s following:
antigens 1. Prodromal period
4. the recipient’s immunoglobulins react 2. Convalescent period
with donor’s erythrocytes 3. Incubation period
4. Lag period
109. The distinguishing characteristics of from the primary response by having
infectious disease are which of the which of the following:
following: It 1. a shorter latent period
1. is caused by pathogenic microbe 2. a longer duration of production of
2. is communicable antibodies
3. is followed by a specific immune 3. a greater quantity of antibodies
response in the host produced
4. can be caused by insects 4. the prevalence of Ig M antibodies
110. Virulence factors of microbes can be
groupped as follows: Factors that 117. The MHC (or HLA) class I and II
1. favor establishment of infection proteins share which of the following:
2. reduce microbial pathogenicity They are both
3. promote pathogen survival and host 1. integral membrane proteins
injury 2. serve to restrict responses of T-killers
4. protect host tissue from damage 3. codominantly expressed on cells
111. The antiphagocytic virulence factors are 4. expressed on all nucleated cells
which of the following: 118. Which of the following statements are true? Serum
1 antichemotactic factors sickness occurs if:
2. peroxidase 1. anti-basement membrane antibodies
3. capsule are present
4. β-lactamase 2. extreme excess of antibody develops
112. Which of the following functions have 3. Ig E antibodies are produced
both a primary and secondary lymphoid 4. soluble immune complexes are
organs? formed
1. Cellular proliferation 119. T-cell-mediated immune response can
2. Differentiation of lymphocytes result in:
3. Cellular interaction 1. serum sickness
4. Antigen-dependent response 2. formation of granuloma in internal
113. Which of the following statements organs
characterize haptens? They 3. anaphylaxis
1. require carrier molecules in order to 4. transplant rejection
be immunogenic 120. CD4+ and CD8+ T-cells in humans differ
2. will not react with specific antibodies from each other by which of the
in vitro unless homologous carriers following:
are employed 1. function
3. interact with specific antibody 2. restriction elements encoded by the
4. can stimulate secondary antibody MHC
response without carriers 3. production of B-cell-activating
114. Complement is required for lysis of: lymphokines
1. bacteria by specific antibodies 4. proliferative response to IL-2
(bacteriolysins) 121. Blood from group O donors can be
2. bacteria by bacteriophages transfused to a recipient without causing a
3. erythrocytes by specific antibodies transfusion reaction if:
(hemolysins) 1. the recipient is A
4. erythrocytes by lecithinase 2. the recipient is B
115. Immunoglobulins that activate the first 3. the recipient is AB
component of complement (C1q) via the Fc 4. the recipient is O
portion include which of the following: 122. Transplant rejection may involve which
1. Ig G of the following immune processes:
2. Ig A 1. type IV delayed hypersensitivity
3. Ig M 2. cell-mediated immunity
4. Ig E 3. α - IFN releasing
116. The secondary antibody response differs 4. complement-dependent cytotoxicity 
123. Intentional protection of a future 3. variable regions
newborn against Rh disease involves 4. constant regions
which of the following? 130. Which of the following are the properties
1. Active immunization of the mother to of human Ig G:
produce transplacental Ig G 1. It can pass through placenta
2. Passive immunization of the newborn 2. It can be cleaved by pepsin and still
to remove the Rh antigen remains divalent
3. Active immunization of the mother 3. It is the main Ig of secondary immune
with Rh antigen to induce antibody response
4. Passive immunization of the mother 4. It induces the formation of leukocytes
to remove Rh antigen 131. Which of the following statements are
124. Which of the following statements true? The alternative pathway of
characterize the pathogenicity of complement activation is characterized
microbes? by the:
1. It is a potential ability to cause 1. activation of complement components
infectious disease beyond C3 in the cascade
2. It depends on environmental factors 2. participation of properdin
3. It is genetically encoded microbial 3. generation of anaphylatoxin
species feature 4. utilization of C4
4. It can be increased or decreased 132. Which of the following statements
within the host organism characterize interferon-γ? It
125. The nonspecific cell-mediated factors of 1. is synthesized by macrophages
host defense are which of the following: 2. is released as a result of antigen- or
1. complement mitogen-induced activation of T-
2. T-lymphocytes lymphocytes
3. normal microbial flora 3. binds specifically to the antigen that
4. phagocytosis induces its releasing
126. Which of the following features apply to 4. induces macrophages to ingest and
bacterial exotoxins? They are destroy bacteria in a nonspecific
1. highly immunogenic fashion
2. converted into toxoid
3. specific in their mode of action
4. proteins
127. Which of the following are the steps of 133. Thymus processing is not necessary for
phagocytosis? maturation of which of the following
1. Chemotaxis cells:
2. Attachments 1. helper T- cells
3. Ingestion 2. B-cells
4. Phagolysosome formation 3. cytotoxic T- cells
4. monocytes
134. Which of the following statements
128. Which of the following are the functions concerning T- and B-cell cooperation are
of secondary lymphoid organs? true?
1. antigen-dependent response 1. Helper T-cells produce soluble
2. circulation of lymphocytes factors that induce B-cells and T-
3. terminal differentiation of immune killers to proliferate
cells 2. Helper T-cells recognize carrier
4. cellular proliferation determinants which may differ from
129. The class-specific antigenic determinants the determinants recognized by the
(isotypes) of immunoglobulins are B-cells
associated with which of the following: 3. Helper T-cells can recognize antigen only
1. L - chains when it is presented in association with MHC
2. H - chains (or HLA) class II molecule
4. Helper T-cells must be in direct because:
contact with B-cells to induce them to 1. the donor’s immunoglobulins react
proliferate with recipient’s erythrocytes
135. CD4+ cells can: 2. a mixed lymphocyte reaction takes
1. help B-cells to make antibody place
2. process and present antigen 3. the recipient’s T lymphocytes will
3. amplify activities of T-cells become activated by the donor’s
4. release IL-1 antigens
136. CD8+ cells kill their targets by which of 4. the recipient’s immunoglobulins react with
the following processes: donor’s erythrocytes
1. recognition of peptides on MHC (or 142. Blood from group B donors can be
HLA) class I molecules transfused to a recipient without causing a
2. activation of the alternative transfusion reaction if:
complement pathway 1. the recipient is B
3. insertion of representative molecules 2. the recipient is O
into target membranes 3. the recipient is AB
4. binding via Fc receptors to antibody - 4. the recipient is A
coated targets UNIT 4. GENERAL VIROLOGY
137. A patient is admitted with multiple Task . Choose the appropriate
bacterial infections and is found to have a answer:
complete absence of C3. Which of the A if only 1, 2 and 3 are correct
following complement-mediated functions B if only 1 and 3 are correct
would remain intact in such a patient? C if only 2 and 4 are correct
1. lysis of bacteria D if only 4 is correct
2. opsonization of bacteria E if all are correct
3. generation of anaphylatoxins
4. none of the above
138. The administration of vaccines is not 1. Which of the following are the
without hazard. Which of the following distinguishing characteristics of viruses?
vaccines is most likely to affect adversely They
an immunocompromised host? 1. require complex nutrient media for
1. Killed vaccine their growth
2. Subunit vaccine 2. lack cell structure
3. Toxoid 3. are capable of independent
4. Live vaccine metabolism
139. Immune complexes are involved in the 4. are obligate intracellular parasites
pathogenesis of which of the following: 2. Viruses can be classified by which of the
1. Farmer’s lung following order?
2. Serum sickness 1. Type and strandedness of nucleic
3. Glomerulonephritis or systemic lupus acid
erythematosus 2. Number of capsomeres
4. Anaphylaxis 3. Presence or absence of envelope
140. Specific nonresponsiveness to self and 4. Symmetry of nucleocapsid
other antigens may be mediated by which 3. The distinguishing characteristics of the
of the following: simple viruses are which of the following?
1. clonal deletion of self-reactive B-cells They are
2. specific suppressor T-cells 1. “defective” viruses
3. clonal deletion of self-reactive T- 2. unprotected by capsid
cells: early stages of maturation 3. viroids
4. rapid digestion and removal of 4. not enveloped
antigen from the circulation 4. The main functions of viral nucleic acid are
141. Blood group incompatibility usually which of the following:
possess a transfusion reaction risk 1. Antigenicity
2. Infectiveness 2. Animals
3. Adsorption to the host cell 3. Chicken embryos
4. Heredity 4. Bacterial cell cultures
12. Which of the following methods can be
employed to study the morphology of
viruses?
5. The stages of “productive” type of viral 1. Phase-contrast microscopy
reproduction circle are which of the 2. Immune electron microscopy
following: 3. Dark-field microscopy
1. Uncoating (deproteinization) 4. Electron microscopy
2. Assembly of progeny viruses 13. The virus detection in the infected cell
3. Virus genome replication cultures is based on which of the following
4. Virus genome integration into the host cell tests:
chromosome with no progeny production 1. Hemadsorption test
6. Which of the following characterize 2. Plaques formation test
«negative»-sensed viral RNA? It 3. Hemagglutination test
1. contains heredity material 4. HAI-test
2. is capable of reversed transcription 14. The tests for virus detection in the infected
3. has no functions of mRNA chicken embryos are which of the
4. is capable of integration into host following:
cell chromosome 1. Color test
7. Antiviral antibodies in paired patient’s sera 2. Hemadsorption test
may be determined by which of the 3. Plaques formation test
following tests: 4. Hemagglutination test
1. CF-test 15. Virus identification may be performed by
2. ELISA which of the following immune tests:
3. HAI-test 1. HAI-test
4. IHA test 2. Hemagglutination test
8. The integration of viral nucleic acid into 3. CF- test
the host cell chromosome is reffered to as 4. PCR
which of the following?
1. Conversion
2. Lysogeny
3. Conjugation
4. Virogeny 16. Which of the following statements
9. Which of the following events occur after characterize the ability of virus to escape
bacterial cell penetration by temperate the host cell by budding?
bacteriophage?
1. Prophage formation 1. Escaping by budding is characteristic
2. Lysogeny for the enveloped viruses
3. Integration of viral DNA into 2. This process is called exocytosis
bacterial chromosome 3. The host cell survives
4. Bacterial cell lysis 4. It occurs only in DNA-genome
10. The bacteriophages can be used for which of viruses
the following purposes: 17. The HAI-test is considered to be
1. Bacteriophage typing of bacteria «positive», if:
2. Treatment of infectious diseases 1. the complete hemolysis occurs
3. Genetic engineering 2. the red «button» is formed
4. Cultivation of bacteria 3. the red granular film is detected in
11. Which of the following animated models the well
can be employed for human viruses 4. the specific antibodies inhibit the
culturing: virus ability to agglutinate erythrocytes
1. Eukaryotic cell cultures 18. The VHA-test is considered to be
«positive», if:
1. the virus with hemagglutinins 3. HAI-test
presents in a sample tested 4. ELISA
2. the specific antibodies inhibit the 25. The stages of “integrative” type of virus
virus ability to agglutinate replication cycle are which of the
erythrocytes following:
3. the red granular film is detected in 1. Attachment
the well 2. Virus genome integration into the
4. the red «button» is formed host cell chromosome with no
19. Which of the following are the progeny formation
distinguishing characteristics of viruses? 3. Uncoating (deproteinization)
They 4. Assembly of progeny viruses
1. require enriched culture media for
their propagation
2. lack cellular structure
3. are capable of independent 26. The VN-test in the cell culture is
metabolism considered to be «positive», if
4. are obligate intracellular parasites 1. the virus cytopathic effect is
20. Which of the following characterize neutralized by specific antibodies
«positive»-sensed viral RNA? It 2. yellow color is observed in the tube
1. contains hereditary material with cell culture
2. has internal antigens 3. the inoculated cell culture remains
3. has functions of m-RNA alive after specific antibodies adding
4. is capable of integration into the host 4. the virus ability to cause
cell chromosome hemagglutination is inhibited
27. The non-specific factors of the host
defense against viral infections are which
of the following:
1. Serum inhibitors
21. The bacteriophages can be employed for 2. Ig G
which of the following purposes: 3. Interferons
1. Typing of bacteria 4. Ig M
2. Treatment of infectious diseases 28. The VN-test in the cell culture is
3. Genetic engineering considered to be «positive», if
4. Prophylaxis of infectious diseases 1. the viral cytopathic activity is neutralized by
22. The tests for virus detection in the infected specific antibodies
cell cultures are which of the following: 2. the toxins are neutralized by specific
1. inclusion bodies formation antibodies
2. giant cells formation 3. the inoculated cell culture is alive after addition
3. destruction of monolayer of specific antibodies
4. plaques formation test 4. the virus ability to produce
23. Which of the following events may occur hemagglutination is inhibited
after bacterial cell penetration by 29. The non-specific factors of the host
temperate bacteriophage? defense against viral infections are which
1. Prophage formation of the following:
2. Lysogeny 1. Serum inhibitors
3. Integration of viral DNA into 2. T-lymphocytes
bacterial chromosome 3. Interferons
4. Bacterial cell lysis 4. Ig A
24. Antiviral antibodies in a patient’s serum 30. Which of the following are the distinguishing
can be determined by which of the characteristics of viruses? They
following tests: 1. are obligate intracellular parasites
1. CF-test 2. possess both DNA and RNA
2. IHA test 3. are incapable of independent
metabolism 4. Virus hemagglutination test
4. require enriched nutrient media for 38. The HAI-test is considered to be
their propagation «positive», if:
31. Viruses can be classified by which of the 1. the red «button» is formed
following order? 2. the red granular film is detected in
1. Type and strandedness of nucleic the well
acid 3. the specific antibodies inhibit the virus ability to
2. Number of capsomeres agglutinate erythrocytes
3. Presence or absence of envelope 4. the complete hemolysis occurs
4. Symmetry of nucleocapsid 39. The VN-test in the cell culture is
considered to be «positive», if
32. The specific factors of antiviral immune response are 1. the virus cytopathic effect is
which of the following: neutralized by specific antibodies
1. Phagocytosis 2. the toxins are neutralized by specific
2. T-lymphocytes antibodies
3. Interferons 3. the inoculated cell cultureremains alive after
4. Antibodies (Ig M, Ig G, Ig A) specific antibodies adding
33. The distinguishing characteristics of the 4. the virus ability to cause
naked viruses are which of the following: hemagglutination is inhibited
They are 40. The non-specific factors of the host
1. “defective” viruses defense against viral infections are which
2. unprotected by capsid of the following:
3. viroids 1. Serum inhibitors
4. not enveloped 2. T-lymphocytes
34. Which of the following statemnets 3. Interferons
characterizes «negative»-sensed viral 4. Ig G
RNA? It 41. Which of the following statements concerning
1. contains hereditary material complex viruses are true?
2. is capable of reversed transcription 1. They are cultured on highly enriched
3. has no functions of mRNA culture media
4. is capable of integration into host 2. They possess both DNA and RNA
cell chromosome 3. They are unprotected by capsid
35. Which of the following animated models 4. They have envelope
can be employed for human viruses 42. The stages of “productive” type of
culturing? bacteriophage reproduction in bacterial
1. Eukaryotic cell cultures cell are which of the following:
2. Animals 1. Injection of viral DNA into the host
3. Chicken embryos cell
4. Bacterial cell cultures 2. Incorporation of bacteriophage DNA
36. Which of the following methods can be into bacterial genome
employed to study the morphology of 3. Assembly of progeny bacteriophages
viruses? 4. Exocytosis
1. Phase-contrast microscopy 43. Which of the following statements
2. Immune electron microscopy characterizes the ability of virus to escape
3. Dark-field microscopy the host cell by budding?
4. Electron microscopy 1. Enveloped viruses escape the host
37. The tests for virus detection in the infected cell by budding
chicken embryos are which of the 2. This process is called exocytosis
following: 3. The host cell survives
1. Color test 4. Budding occurs mainly in
2. Pocks on CAM (chorionallantoic bacteriophages
membrane) 44. Which of the following are the
3. Hemadsorption test distinguishing characteristics of viruses?
They 52. The virus identification can be performed
1. require living models for their by which of the following tests:
replication 1. HAI -test
2. have no cellular structure 2. CF - test
3. are incapable of independent 3. VN - test
metabolism 4. VHA - test
4. are facultative intracellular parasites 53. The VHA-test is considered to be
45. Viruses can be classified by which of the «positive», if
following order: 1. complete hemolysis occurs
1. Type of nucleic acid 2. the virus ability to produce
2. Strategy of genome hemagglutination is inhibited
3. Number of chains of nucleic acid 3. the virus is neutralized by specific
4. Shape and size of virus antibodies
46. The stages of “productive” type of viral 4. the virus causes clumping of
replication cycle are which of the erythrocytes
following: 54. The specific factors of antiviral immune
1. Attachment response are which of the following:
2. Uncoating (deproteinization) 1. Phagocytosis
3. Virus genome replication 2. T-lymphocytes
4. Assembly of progeny viruses 3. Interferons
47. Which of the following events may occur 4. Ig G
after bacterial cell penetration by virulent 55. The main features of viral nucleic acid are
bacteriophage? which of the following: It
1. Prophage formation 1. contains antigens
2. Lysogeny 2. is RNA or DNA
3. Integration of viral DNA into 3. is responsible for adsorption to the
bacterial chromosome host cell
4. Bacterial cell lysis 4. contains hereditary material
48. The tests for virus detection in the infected 56. The stages of “productive” type of viral infection are
chicken embryos are which of the which of the following:
following: 1. Uncoating (deproteinization)
1. Color test 2. Assembly of progeny viruses
2. Hemadsorption test 3. Virus genome replication
3. Hemagglutination inhibition test 4. Virus genome integration into the
4. Virus hemagglutination test host cell chromosome with no
49. Which of the following immune tests progeny formation
allow to identify virus? 57. The integration of bacteriophage DNA
1. HAI-test into the bacterial cell chromosome is
2. Virus hemagglutination test reffered to as which of the following:
3. CF- test 1. Conversion
4. PCR 2. Virogeny
50. Which of the following living models can 3. Conjugation
be employed for viruses culturing: 4. Lysogeny
1. Animals 58. The bacteriophages can be used for which of
2. Eukaryotic cell cultures the following purposes:
3. Chicken embryos 1. phagotyping of bacteria
4. Bacterial cell cultures 2. treatment of infectious diseases
51. The virus detection in the infected animal 3. genetic engineering
is based on which of the following tests: 4. culturing of bacteria
1. Hemadsorption test 59. The virus detection in the infected cell
2. Virus hemagglutination test cultures is based on which of the
3. Plaques formation test following tests:
4. Clinical signs 1. Hemadsorption test
2. Plaques formation test Match each of the following diagnostic laboratory
3. Virus hemagglutination test test with its characteristic:
4. Virus cytopathic effect 80. HA-test (VHA-test) A. It is performed for virus
60. Antiviral antibodies in paired patient’s 81.Hemadsorption test inoculated cell cultures
sera can be determined by which of the 82.HAI-test B. It is used for virus
following tests: 83.Color test inoculated laboratory anim
1. CF-test 84.VN-test C. Both A and B
2. VN-test D. Neither A no B
3. HAI-test Match each of the following diagnostic laboratory
4. VHA-test test with its characteristic
61. The specific factors of antiviral immune response are 85. CF-test A. It is used for ide
which of the following: 86. IF-test (direct method) viruses
1. Phagocytosis 87. HAI-test B. It is used for determi
2. T-lymphocytes 88. HA-test (VHA-test) antibody titer rise
3. Interferons C. Both A and B
4. Antibodies (Ig M, Ig G, Ig A) D. Neither A no B
Match each of the following term listed below Match each of the following diagnostic laboratory
with its definition: test with its characteristic
62. Prions A. Mature viral particles 89.HA-test (VHA-test) A. It is employed for det
63. Viroids B. Uncoated infectious circulated RNA molecules 90.IF-test (direct method) antibodies raising
64. Virions C. Infectious protein particles 91.CF-test B. It is employed for
D. Cytoplasmic inclusion bodies viruses in the specime
Match each of the term listed below with its C. Both A and B
definition: D. Neither A no B
65. Prions A. Mature viral particles Match each of the following characteristic
66. Viroids B. Cytoplasmic inclusion bodies with the appropriate diagnostic laboratory
67. Virions C. Infectious protein particles test:
D. Uncoated infectious circulated RNA 92.Virus detection in inoculated chicken embryos
molecules A.
Match each of the following scheme of viral 93.Virus detection in inoculated cell cultures B.
proteins synthesis listed in right column with 94.Virus identification C.
the appropriate strategy of viral genome: Ne
68. Single-stranded positive-sensed A. DNAà mRNA Match each of the following cell culture line
RNA B. RNAàprotein with its characteristic:
69. Single-stranded negative-sensed C. RNAàmRNA 95.Continuous cell lines A. Only 1-2 generation
RNA D. RNAà 96.Diploid cell lines B. Cells are capable
70.Double-stranded RNA tein 97.Primary cell cultures indefinite growth
(Retroviruses) C. Both A and B
71. Double-stranded DNA D.Neither A no B
Match the characteristics listed below with the
appropriate diagnostic laboratory test: Match each of the following strategy of viral
genomecell
72.HA-test (VHA-test) A. It is used for virus identification in inoculated listed
culturebelow with the appropriate
73.HAI-test B. It is employed for determining antiviral antibody titerproteins synthesis:
scheme of viral
74.VN-test rise 98. Single-stranded A. DNA à mRNA à pro
C. Both A and B positive-sensed RNA B. RNA à protein
D. Neither A no B 99. Single-stranded C. RNA à mRNA à pro
Match each of the following diagnostic negative-sensed RNA D. RNA à DNA à mRN
laboratory test with its characteristic: 100. Double-stranded RNA E. DNA à protein
75. VHA-test (HA-test) A. It is performed to detect virus(Retroviruses)
in
76. Hemadsorption test inoculated cell cultures 101. Double-stranded DNA
77. HAI-test B. It is used for virus detection in inoculated
78. Color test laboratory animals Match each of the term listed below with its
79. VN-test C. Both A and B definition::
D. Neither A no B 102.Prions A. Mature viral particles
103.Viroids B. Uncoated infectious circulated RNA molecules
104.Virions C. Infectious protein particles
D. Cytoplasmic inclusion bodies Match each of the following cell culture line
with its characteristic:
Match each of the following diagnostic 110. Continuous cell A. They remain viable for 1-
laboratory test with its characteristic: cultures only
105. CF-test A. Is used for identification111. Diploid cell cultures B. Cells are capable for pro
of viruses
106. IF-test (direct method) B. Is used for determination 112.
of Primary cell indefinite growth
107. HAI-test antiviral antibodies titer cultures C. Both A and B
108. VHA-test C. Both A and B D. Neither A no B
109. VN -test D. Neither A no B
The EXAMPLE of the Examination CASE
(the names of pathogens can be found in the “Topics for Microbiology Exam”,
Special Bacteriology and Special Virology, see above!)
CASE
A 35-year-old patient with fever, abdominal pain, and diarrhea was
admitted to the hospital. The patient was diagnosed to have shigellosis.
1. Choose the appropriate card to characterize the causative agent of bacterial
dysentery.
2. Fill in the card with the characteristic features of bacterial dysentery causative
agent.

EXAMINATION CARD

Task: Characterize the causative agent of bacterial infection according to the scheme:

1. Taxonomy: Family__________________ Genus_________________ Species___________


2. Morphology (shape, main structures, appendages) and tinctorial
properties___________________________________________________________________
3. Cultural properties Respiration type_________, culture media______________________,
time of exposure ______, temperature_______, growth characteristics____________________
4. Biochemical activity_________________________________________________________
5. Antigenic structure__________________________________________________________
6. Virulence factors___________________________________________________________
7. Resistance (to adverse factors)_________________________________________________
8. Epidemiology sources of infection__________________________________, routes and
modes of transmission________________________________________________________
9. Pathogenesis of infection (portals of entry and exit, main stages of
disease)___________________________________________________________________
10. The main clinical
findings___________________________________________________
11. Immunity after
infection____________________________________________________
12. Microbiological diagnosis
A. Specimens:___________________________________________________________
B. Methods, tests:________________________________________________________
13. Treatment (antimicrobials and immunobiological
preparations)______________________
14. Immunoprophylaxis (immunobiological
preparations)_____________________________
The EXAMPLE of the Examination CASE

CASE
The symptoms of jaundice developed in the 37-year-old patient 3 months
after blood transfusion. The physician suspects hepatitis B.
2. Choose the appropriate card to characterize the causative agent of hepatitis B.
2. Fill in the card with the characteristic features of hepatitis B causative agent.

EXAMINATION CARD

Task: Characterize the causative agent of viral infection according to the scheme:

1. Taxonomy and Classification: Family_______________ Genus____________________


Virus name_________________________________________________________________
2. Morphology (shape, size (small, medium, large), naked/enveloped____________________
3. Genome (Point out: RNA/DNA, single stranded/double stranded, positive sense, negative
sense)____________________________________________________________________
4. Culturing (sensitive models, indication tests)_____________________________________
5. Antigenic structure_________________________________________________________
6. Resistance (to adverse factors)________________________________________________
7. Epidemiology sources of infection________________________, routes and modes of
transmission_______________________________________________________________
8. Pathogenesis of infection (portals of entry and exit, main stages of
disease)___________________________________________________________________
9. The main clinical findings___________________________________________________
10. Immunity after infection____________________________________________________
11. Microbiological diagnosis
A. Specimens:___________________________________________________________
B. Methods, tests:________________________________________________________
12. Treatment (antimicrobials and immunobiological preparations)______________________
13. Immunoprophylaxis (immunobiological preparations)_____________________________
The EXAMPLES of the Examination Tasks

TASK
Pure bacterial culture was isolated from the blood of a patient suspected of having typhoid
fever. Bacteria tested were gram-negative rods which formed colorless colonies on the Endo agar
(Photo will be given). The colorless colonies were subcultured onto Kligler iron agar Simmons’
citrate medium.
QUESTIONS:
1) Explain the results of bacterial growth on the Kligler iron agar.
2) Explain the results of bacteriological examination.
3) What causative agent was isolated from tested blood ?
4) What should be next steps of bacteriological examination to prove the clinical diagnosis?
5) How microbiological culture media can be classified? Characterize differential media. Give
an example of differential media.
TASK
A patient with fever, abdominal pain, and diarrhea was admitted to the hospital. The patient
was diagnosed to have a bacterial dysentery. Lactose-negative and gram-negative rods were
isolated from the patient’s mucus-impregnated feces.
QUESTIONS:
1) Look at Photo 1 and point the colonies on Endo agar which are characteristic of lactose-
negative bacteria.
2) Explain the results of bacterial growth in Kligler iron agar and mannitol-containing Hiss’
medium (Photo will be given).
3) What causative agent was isolated from tested specimen?
4) What steps should be done next to prove the clinical diagnosis?
5) What steps does bacteriologic examination consist of?

TASK
A 27-year-old businessman had nausea, vomiting and bloody stools for two days. Then he
developed renal failure. Patient’s stool specimen was inoculated onto Endo agar. Photo 1 shows
the results of bacterial growth on Endo agar.
QUESTIONS:
1) Explain the results of bacterial growth on Endo agar. What enteric pathogen was isolated in
this case?
2) What is the main virulence factor of this pathogen?
3) What steps should be made next to prove the diagnosis bacteriologically?
4) Explain the results of bacterial growth on Kligler iron agar and mannitol-containing Hiss’
medium (Photo will be given).
5) What is the E. coli role in human normal flora and in the development of infectious
processes?
TASK
A young adult has come to the emergency room with complaints of fever, severe cough,
and pain in the chest. The physician suspects pneumonia.
QUESTIONS:
1) What pathogens can cause pneumonia?
2) What specimen should be taken from the patient? Describe the procedures of collection and
transport of the material taken.
3) How can physician identify specific organism that causes the infection? Describe the
procedure of patient’s specimen examination and results evaluation.
4) Explain the term “nosocomial infections”.
5) What inoculation techniques do you know?
TASK
A 46-year-old traveler from Russia visited India for 2 weeks. Two days after arrival back
he complained nausea, vomiting, watery diarrhea and he was admitted to a hospital. A stool
specimen was taken and examined microscopically. Gram staining revealed the great amount of
gram-negative comma-shaped bacteria in stools. The patient was diagnosed to have cholera.
QUESTIONS:
1) What is the name of cholera causative agent? How cholera pathogens can be classified?
2) What bacteriological laboratory tests can be used to confirm the diagnosis of cholera?
3) What are the tests for rapid diagnostics?
4) What are the virulence factors of cholera pathogen?
5) What vaccines are available for cholera immunoprophylaxis?

TASK
An 8-year-old child is admitted to a hospital with diagnosis diphtheria. The throat swab is
examined microscopically and rods in angular patterns that look like Chinese letters with volutin
granules in their ends are observed.
QUESTIONS:
1) Name the diphtheria pathogen. What is the main virulence factor of this pathogen?
2) What other specific tests have to be used to identify the diphtheria pathogen?
3) What are the immune biological preparations that can be used for treatment and
immunoprophylaxis of diphtheria?
4) What categories of acquired humoral immunity do you know?
5) How to prepare toxoid?

TASK
A patient complained of severe diarrhea that had begun the previous morning. The whole
family turned out to be sick (two adults and three children, but not the baby). They all had had
traditional dinner consisting of roast turkey with stuffing, cranberry sauce, sweet potato
casserole, and custard pie. From the patient’s feces gram-negative and lactose-negative rod-
shaped bacteria were isolated. They produced black colonies on the bismuth-sulfite agar.
QUESTIONS:
1) What pathogens may be suspected as the cause of diarrhea?
2) What are the routes and modes of disease transmission?
3) How can the etiologic agent be identified?
4) What treatment should be recommended?
5) Explain the term “taxonomy”. What is the taxonomy and classification of salmonellae?

TASK
You are employed in a public health laboratory. You have managed to isolate Salmonella
from both the egg salad and the chicken salad served at a company picnic after which 5 people
became ill.
QUESTIONS:
1) What laboratory procedures were used to isolate the microbes?
2) What special tests are necessary to be performed to identify Salmonella?
3) What laboratory procedures (including “genetic tests”) can be used to track down the source
of infection?
4) Characterize bacteriophages. What types of bacteriophages do you know?
5) Explain the term “lysogeny”.
TASK
Dr. Adler is the chief of the laboratory in University Hospital, where the outbreak of
wound infections occured. Laboratory personnel isolated S. aureus from 12 patient’s specimens.
Nasopharyngeal cultures from the nurses and physicians yielded another 5 S. aureus isolates. Dr.
Adler wants to know if any member of the staff is the carrier of the pathogen.
QUESTIONS:
1) What culture media should be used to obtain S. aureus pure culture?
2) Explain the term “pure culture”.
3) What tests should be performed to track down the source of infection?
4) What tests are usually performed to identify S. aureus?
5) What specific tests can be used to choose the most effective antibacterial drugs for treatment
of infections?

TASK
A patient with diarrhea (a «rice water» stools appearance) and severe dehydration was
admitted to a hospital. He was diagnosed to have a cholera.
QUESTIONS:
1) What are the main virulence factors of the pathogen?
2) What is the mechanism of V. cholerae toxin action?
3) What vaccines are available for cholera immunoprophylaxis?
4) What tests should be performed to distinguish between “pathogenic” and “non-pathogenic”
vibrios?
5) Explain the term “lophotrichous” How to detect motility of bacteria?

TASK
Three patients were admitted to the local emergency room with leg traumas and wounds.
Arlene, 76 year-old man, had stepped on a nail while working in a garden. Antitoxic
tetanus immunoglobulin was given to him for tetanus immunoprophylaxis.
7 year-old Aaron had fallen down from his bike and scraped himself. Tetanus toxoid was
administered to him.
Dave, 27 year-old builder, with shoulder fracture was treated with tetanus toxoid and
antitoxic tetanus immunoglobulin.
QUESTIONS:
1) What virulence factor of tetanus pathogen may cause the symptoms of the disease? What is
the mechanism of its action?
2) Characterize tetanus toxoid and antitoxic tetanus immunoglobulin.
3) What categories of acquired humoral immunity do you know?
4) The development of what type of immunity (active or passive) causes the antitoxic tetanus
immunoglobulin given? The tetanus toxoid?
5) Explain the doctor’s tactics for each patient treatment.
TASK
Goat’s wool was tested in precipitin ring test for B. anthracis antigen detection (Ascoli’s
test). The result of reaction is shown on Photo.
QUESTIONS:
1) Look at the picture and estimate the results. Make a conclusion.
2) What is the mechanism of precipitin ring test?
3) What are the possible mechanisms for anthrax transmission?
4) What preparations are available for treatment and immunoprophylaxis of anthrax?
5) Characterize endospores of bacteria. What staining procedures help to detect their presence?
TASK
The symptoms of jaundice developed in the patient 3 months after blood transfusion. The
HBs antigen was detected in the patient’s blood serum.
QUESTIONS:
1) What disease should be suspected in this case?
2) What is the causative agent of the disease? What “markers” can confirm acute infection?
2) What laboratory tests can be used to diagnose this infection?
3) What types of antigens does hepatitis B virus possess?
4) Explain the term “antigen”.
TASK
Hospital-acquired infections are often difficult to treat because wide-spread use of
antimicrobial chemotherapy in the hospital encourages the emergence of drug-resistant microbes.
The antibiotic susceptibility test was performed to choose the most effective drugs for treatment.
Look at the picture (Photo) and make a conclusion.
QUESTIONS:
1) Which of the following drugs is the microbe tested mostly sensitive to?
2) Describe the procedure of disk diffusion method.
3) What other methods can be used to test antimicrobial susceptibility of bacterial isolates?
4) What changes in bacterial cell lead to an acquired antibiotic resistance?
5) Explain the term “nosocomial infections”.
TASK
The paired patient’s sera are examined in the Hemagglutination Inhibition test (HAI-test)
to confirm the influenza diagnosis. The results can be seen on Photo.
QUESTIONS:
1) Estimate the results of serologic examination of patient’s serum. Make a conclusion.
2) What serotype of virus caused the disease?
3) What is the mechanism of HAI-test?
4) What other methods can be performed for influenza diagnostics?
5) What are the principles of viruses cultivation?

TASK
A 26-year-old man presented to his family practitioner with a purulent penile discharge.
Gonorrhea was diagnosed based upon the finding of intracellular gram-negative coffee-bean-
shaped diplococci in his discharge.
QUESTIONS:
1) What pathogen was detected in sample tested?
2) What phenomenon was observed in stained smear?
3) What other diagnostic laboratory tests can be used to confirm the diagnosis?
4) What non-specific factors of host defense do you know?
5) What are the steps of phagocytosis? Name the phagocytic cells.
TASK
A patient with symptoms of botulism was admitted to a hospital. The patient’s specimens
(gastric lavage waters) were tested in the Indirect Hemagglutination test (IHA-test). The results
of reaction are shown in Table 1.
QUESTIONS:
1) Estimate the results of IHA-test. Make a conclusion.
2) What is the mechanism of IHA-test?
3) What virulence factor of C. botulinum may cause the symptoms of disease? What is the
mechanism of its action?
4) What immunobiological preparations can be used for botulism treatment?
5) Explain the terms: “pathogenicity” and “virulence”.
TASK
A 32-year-old patient with symptoms of viral keratitis came to physician. The swab from
conjunctiva was sent to the laboratory. The virusologic examination was performed and
Adenovirus with unknown serotype was isolated. The results of the Virus-Neutralization Test
(VN-test) in cell culture for virus identification are presented on Photo 1.
QUESTIONS:
1) What is the mechanism of VN-test?
2) Look at the picture and explain the results. Make a conclusion.
3) What other tests can be used to identify viruses?
4) What are the steps of virusologic examination?
5) What are the distinguishing characteristics of viruses?

TASK
Pure bacterial culture was isolated from the blood of a patient suspected of having typhoid
fever. Bacteria tested were gram-negative rods which formed colorless colonies on the Endo
agar. The colorless colonies were subcultured onto Kligler iron agar Simmons’ citrate medium.
The results of bacterial growth are shown in Photo 1.
QUESTIONS:
1)Explain the results of bacterial growth on the Kligler iron agar.
2)What causative agent was isolated from tested blood ?
3) What should be next steps of bacteriological examination to prove the clinical diagnosis?
4) What is the taxonomy and classification of salmonellae?
5) What is the mechanism of slide agglutination test?

TASK
A patient with fever, abdominal pain, and diarrhea was admitted to the hospital. The patient
was diagnosed to have a shigellosis. Lactose-negative and gram-negative rods were isolated from
the patient’s mucus-impregnated feces.
QUESTIONS:
1) Look at Photo and explain the results of bacterial growth in Kligler iron agar and mannitol-
containing Hiss’ medium (see Photo).
2) What causative agent was isolated from tested specimen?
3) What virulence factors possess shigellae?
4) What group of E.coli (EPEC, ETEC, EIEC or EHEC) causes the same clinical symptoms?
5) How to distinguish between shigellae and E.coli?

TASK
Two groups of experimental animals have been infected by two different strains of E.coli.
The LD50 of E.coli strain M 17 is 5x109 CFU/ml; the LD50 of E.coli strain Hb 101 is 2x108
CFU/ml.
QUESTIONS:
1) Which of the following strains of E.coli is more virulent?
2) Explain the terms: “virulence” and “pathogenicity”.
3) How virulence factors of bacteria can be classified?
4) What virulence factors of bacteria makes them resistant to phagocytosis?
5) What non-specific factors of host defense do you know?
TASK
The nasopharyngeal aspirates specimen of the patient suspected of having influenza was
inoculated into allantoic cavity of chicken embryo. The allantoic fluid was examined in Virus
Hemagglutination Test (VHA-test). The result of reaction can be observed on Photo.
QUESTIONS:
1) Look at the picture and estimate the VHA-test results.
2) What is the mechanism of VHA-test?
3) What steps of virusologic method should be performed next to prove the influenza diagnosis?
4) What other diagnostic methods can be used for influenza diagnostics?
5) What steps of replication cycle of viruses do you know?

TASK
The patient with symptoms of jaundice is admitted to the hospital. The patient’s blood
serum is examined serologically.
The results obtained are as follows:

“Marker” Result
anti-HAV antibodies Ig M +
HBs antigen -
anti-HBc- antibodies Ig M -
anti-HBc- antibodies Ig G -
anti-HBs -antibodies Ig G -
QUESTIONS:
1) What disease should be suspected in this case?
2) What is the causative agent of the disease ? How it is classified.
3) What hepatitis viruses do you know?
4) What are the routes and modes of their transmission?
5) What laboratory tests can be used to diagnose hepatitis A and E?

TASK
The 54-year-old insulin-dependent diabetic woman was admitted to a hoepital with a hot,
painful ulcer of right foot. The surgeon suspects gas gangrene and the wound material was sent
to laboratory.
QUESTIONS:
1) What pathogens can cause gas gangrene?
2) What virulence factors they possess?
3) What are the proper conditions for collection, transport and culturing of anaerobic bacteria?
4) What immune biological preparations can be used for gas gangrene treatment and
immunoprophylaxis?
5) What “respiration types” of bacteria do you know?
TASK
Two groups of experimental animals have been infected by two different strains of
Klebsiella pneumoniae.. The LD50 of K.pneumoniae strain A is 109 CFU/ml; the LD50 of
K.pneumoniae strain B is 107 CFU/ml.
QUESTIONS:
1) Which of the following strains of K.pneumoniae is more virulent?
2) Explain the terms: “pathogenicity” and “virulence”.
3) How virulence factors of bacteria can be classified?
4) What virulence factors that favor colonization and infection spreading do you know?
5) What is the role of resident normal flora in host defense?
TASK
A 37-year-old woman visited her general practitioner complaining of a burning sensation
when passing urine, which was more frequent than usual. The general practitioner chose to wait
for the results of urine culture before prescribing any antibiotics.
QUESTIONS:
1) How the urine tested should be inoculated for culture?
2) Describe the procedure of inoculation.
3) What microbiological culture media should be used?
4) Explain the term “opportunistic microorganism”.
5) How to evaluate the results of bacteriological examination?
TASK
A 19-year-old drug addict turns up to the antenatal clinic in an advanced state of
pregnancy. She is very worried because her boyfriend has just told her that he has AIDS.
QUESTIONS:
1) Characterize HIV and its genome.
2) What is the HIV antigenic structure?
3) What laboratory tests should be performed to prove HIV-infection?
4) What are the routes and modes of HIV transmission?
5) What are the steps of replication cycle of viruses?
TASK
A 25-year-old man presented with an infected dog bite. The accident occurred 5 days
previously. The patient is at risk of rabies.
QUESTIONS:
1) What are the distinguishing characteristics of rabies virus?
2) What specimens should be examined for virus detection and identification?
3) What microbiologic laboratory tests should be performed?
4) What are the immune biologic preparations for rabies prevention?
5) What tests can be employed for virus detection (indication) in cell cultures?
TASK
A 6-month-old girl with staphylococcal impetigo (skin infection) is brought into the
pediatric ward. The pus from the pustules is sent to the laboratory. Gram-positive cocci arranged
in a groups resemble bunches of grape are observed by direct microscopic examination.
QUESTIONS:
1) What diagnostic laboratory tests should be used to prove staphylococcal etiology of impetigo?
2) What culture media should be inoculated to isolate staphylococci pure culture? What tests
should be used to identify S. aureus?
3) What virulence factors staphylococci may possess?
4) What infectious diseases they can cause?
5) What are the mechanisms of staphylococci resistance to the ß-lactam antibiotics?
TASK
An 18-month-old child was brought to his general practitioner by his mother who said he
was ‘not well’. The GP noted a temperature of 38 oC, neck stiffness and a purpuric rash over the
body. Acute bacterial meningitis was suspected. An urgent lumbar puncture was performed and 2
ml of purulent cerebrospinal fluid was sent to laboratory. Gram-negative coffee-bean shaped
cocci were detected microscopically.
QUESTIONS:
1) What was the most probable causative agent?
2) What are the virulence factors of a pathogen?
3) What diagnostic laboratory tests should be performed to confirm clinical diagnosis?
4) What vaccines are available for immunoprophylaxis of bacterial meningitis?
5) Explain the term “generalized infection”. What types of generalized infection do you know?

Potrebbero piacerti anche